Вы находитесь на странице: 1из 51

Practice Questions 1

C1 C3 1
A patient has developed congestive heart failure after experiencing his
first myocardial infarction. The pulmonary signs and symptoms the
therapist expect to find include:
A. crackles and cough.
B. inspiratory wheezing and shortness of breath.
C. cough productive of thick yellow secretions.
D. crackles and clubbing of the digits.

C2 C8 1
A newborn who is 4 weeks premature is examined at birth using the
APGAR test. Based on the following results, the neonatal therapist
suspects neurological complications are likely with an APGAR of:
A. 3 at 10 minutes.
B. 8 at 5 minutes.
C. 9 at 1 minute.
D. 8 at 1 minute.

C3 C7 3
A physical therapist requested that a PTA perform ultrasound on the
left shoulder of a patient. During the treatment session, the patient
experienced an electrical shock. The physical therapist would not be
responsible for any injury to the patient if this was the result of:
A. faulty circuitry.
B. the patient touching the ultrasound machine.
C. failure of the PTA to use a ground fault interrupter.
D. failure of the PTA to use sufficient ultrasound gel.

C4 C6 3
A 55 year-old individual with documented coronary artery disease but
no history of myocardial infarction is enrolled in an exercise class that
utilizes circuit-interval training. The MOST appropriate spacing of
work-rest intervals to stress the aerobic system is:
A. 1:1.5
B. 1:7
C. 1:10
D. 1:4.5

C5 C1 3
The BEST INITIAL intervention to improve functional mobility in an
individual with a stable humeral neck fracture is:
A. pendulum exercises.
B. active resistive ROM.
C. isometrics for all shoulder musculature.
D. modalities to decrease pain.

C6 C1 3
A dancer with unilateral spondylolysis at L4 is referred for physical
therapy. She complains of generalized low back pain when she stands
longer than one hour. Interventions for the subacute phase should
include strengthening exercise for the:
A. multifidi working from full flexion back to neutral.
B. abdominals working from neutral to full flexion.
C. multifidi working from neutral to full extension.
D. abdominals working from full extension to full flexion.

C7 C1 1
A patient has fixed forefoot varus malalignment. Possible
compensatory motion(s) or posture(s) might include:
A. excessive subtalar pronation.
B. genu recurvatum.
C. ipsilateral pelvic external rotation.
D. hallux varus.

478026779.doc
2 Practice Questions

C8 C7 3 represents the MOST common risk factor associated with falls in the
The purpose of ABC Physical Therapy Center is to deliver excellent elderly is:
health care services in a unique, individualized and participative A. turning around and sitting down in a chair.
manner at a reasonable cost. The provision of services under these B. climbing on a stepstool to reach overhead objects.
conditions requires and encourages creative involvement of C. walking with a roller walker with hand brakes.
physicians, staff, patients and their agents in all aspects of the care D. dressing while sitting on the edge of the bed.
planning and delivery process. This is an example of a:
A. mission statement. C14 C6 1
B. values statement. A therapist is performing a prosthetic checkout on a patient with a
C. vision statement. transfemoral amputation. The prosthesis has been fitted with a
D. strategic plan. quadrilateral socket. A checkout of the walls of the socket should
reveal that the:
C9 C5 3 A. anterior and lateral walls are 2 1/2 inches higher than
A patient presents with weakness of the knee (2/5) resulting from an the posterior and medial walls.
anterior cruciate ligament injury. The therapist’s examination reveals B. posterior and lateral walls are 2 inches higher than the
moderate pain (5/10) and excessive translation of the tibia during medial and anterior walls.
active knee extension. The therapist determines functional electrical C. height of the posterior wall is 2 inches less than all the
stimulation (FES) is an appropriate intervention. The protocol for other walls.
strengthening the quadriceps and improving stability of the knee D. medial wall is 2 1/2 inches higher than the posterior wall
should consist of stimulation of the: while the anterior and lateral walls are the same height.
A. hamstrings immediately before the quadriceps to produce C15 C7 3
cocontraction. A two month-old child with bilateral hip dislocations is being
B. quadriceps but not the hamstrings. discharged home from an acute pediatric facility. The therapist has
C. hamstrings but not the quadriceps. developed a home exercise program and now needs to instruct her
D. quadriceps immediately before the hamstrings to produce parents. The MOST important item for the therapist to assess before
cocontraction. instructing the parents is:
A. their degree of anxiety and attention.
C10 C2 1 B. the financial reimbursement plan.
A therapist is reviewing a hospital record prior to examining a patient C. their level of formal education.
for the first time. The suspected diagnosis is multiple sclerosis. On the D. the home environment.
neurologist’s note, the therapist finds the following: DTR right
quadriceps is 2+, left quadriceps is 3+. The therapist concludes that: C16 C7 3
A. the right DTR is normal while the left is exaggerated. The grip strength of a group of 50 to 60 year-olds was investigated. A
B. both DTRs are abnormal and indicative of upper motor mean score of 40, SD of 5, and range of 26-57 were reported. The
neuron syndrome. grip strength score for a given patient was determined to be 34. The
C. the right DTR is exaggerated while the left is clearly therapist can safely conclude that in a normal distribution this patient’s
abnormal. score fell within:
D. both DTRs are abnormal and indicative of hyporeflexia. A. 95%.
B. 68%.
C11 C3 1 C. 75%.
A 72 year-old patient has an episode of syncope in the physical D. 32%.
therapy clinic. The therapist attempts to rule out orthostatic
hypotension as the cause of her fainting. This is BEST done by: C17 C3 1
A. checking resting BP and HR in sitting, then repeating Which of the following clinical manifestations is NOT typical of early
measurements after standing for 1 minute. stage cystic fibrosis?
B. checking HR and BP in supine after 5 minutes rest, then A. excessive appetite and weight gain.
repeating in semi-Fowler position. B. increased pulmonary secretions with airway obstruction.
C. palpating the carotid arteries and taking HR; using the C. frequent recurrent respiratory infections.
supine position for BP measurements. D. salty skin and sweat.
D. checking HR and BP at rest, and after 3 and 5 minutes of
cycle ergometry exercise. C18 C8 1
C12 C4 3 An infant demonstrates that the ATNR is NOT obligatory when he/she
An inpatient with a grade III diabetic foot ulcer is referred for physical can:
therapy. Panafil has been applied to the necrotic tissue BID. The A. turn the head and bring the hand to mouth on the same
wound has no foul smell; however, the therapist notes a green tinge side.
on the dressing. In this case, the therapist should: B. turn the head to either side and open the hand.
A. document the finding and continue with treatment C. turn the head to one side and look at the extended arm
B. document the finding and contact the physician on that side.
immediately. D. turn the head to one side and bring the opposite hand to
C. begin a trial of acetic acid to the wound. mouth.
D. fit the patient with a total contact cast.
C19 C4 3
C13 C8 1 A 75 year-old patient is referred to physical therapy for back pain.
The therapist receives a referral to examine the fall risk in an 82 year- Medical tests reveal he is at the end stage of pancreatic cancer. The
old who lives alone and has had two recent falls. The activity that physician has told him he has cancer but has chosen not to inform the

478026779.doc
Practice Questions 3

patient about the prognosis. If the patient asks the therapist what his
prognosis is, the therapist’s BEST response is to tell him:
A. to discuss his concerns with the physician.
B. to ask the nurse practitioner.
C. everything usually works out for the best.
D. the prognosis.

C20 C1 1
During surgery to remove an apical lung tumor, the long thoracic
nerve was injured. Muscle weakness is 3+/5. It would be best to
implement exercises:
A. standing while performing wall push-ups.
B. standing using hand weights.
C. supine using weights.
D. supine using a pulley.

C21 C1 2
A physical therapist examination reveals: iliac crests high on the left;
PSIS low and posterior on the left; ASIS high and anterior on the left;
standing flexion test shows that the left PSIS moves first and farthest
superiorly; Gillet’s test demonstrates the left PSIS moves inferiorly and
laterally less than right; long sitting test shows the left malleolus
moves short to long; sitting flexion test is negative. In light of the
above findings, the therapist’s diagnosis is:
A. left posterior rotated innominate.
B. left anterior rotated innominate.
C. left upslip.
D. iliac inflare on the left.

478026779.doc
4 Practice Questions

C22 C7 3 recently treated with corticosteroids for a bout of myositis.


The highest level of provider risk in reimbursement is related to: Examination findings reveal limited ROM and fibrotic soft tissue along
A. the capitation payment method. with hyperesthesia. The BEST choice for initial intervention is:
B. the cost-based payment method. A. AROM exercises and walking in a therapeutic pool.
C. the fee-for-service payment method. B. soft tissue mobilization and stretching.
D. the per diem payment method. C. closed chain and modified aerobic step exercises.
D. treadmill walking using body weight support at an
C23 C5 3 intensity of 40% HRmax.
A patient presents with partial and full thickness burns on the chest
and neck region. The therapist decides to apply TENS prior to
debridement to modulate pain. Which TENS mode should provide the
BEST relief?
A. brief intense TENS.
B. conventional (high rate) TENS.
C. acupuncture-like (low rate) TENS.
D. modulated TENS.

C24 C2 3
A patient recovering from traumatic brain injury demonstrates
instability during feeding while sitting in a wheelchair. The therapist
determines modification is necessary to ensure optimal function. The
FIRST body segment or segments the therapist would align is the:
A. pelvis.
B. head.
C. lower extremities.
D. trunk.

C25 C2 2
A patient demonstrates beginning recovery Stage 4 movements
following a left CVA. The PNF pattern that represents the BEST
choice to promote continued recovery of the right upper extremity
through the use of out-of-synergy movements is:
A. chop, reverse chop with right arm leading.
B. bilateral symmetrical D1 thrust and reverse thrust.
C. lift, reverse lift with right arm leading.
D. bilateral symmetrical D2F and D2E, elbows straight.

C26 C3 2
A 50 year-old individual has limited endurance as a result of a
sedentary lifestyle. There is no history of cardiorespiratory problems.
Following an exercise tolerance test, which was negative, appropriate
INITIAL exercise prescription parameters for this individual would be:
A. 60 - 90% HRmax
B. 30 - 60% HRmax
C. 40 - 60% HRmax
D. 45 - 55% of VO2max

C27 C3 2
A patient is admitted to a coronary care unit with a mild myocardial
infarction, After 2 days the patient is referred to physical therapy for
exercise conditioning. During an initial exercise session on the unit, he
reports chest pain, appears anxious, and wants to go back to bed to
rest. The therapist’s BEST course of action is to terminate the
exercise and:
A. have him sit down and continue to monitor his vital signs
carefully.
B. monitor his vital signs and contact his doctor
immediately.
C. assist the patient back to bed and contact the charge
nurse on his floor.
D. assign the physical therapist assistant to assist him back
to his bed and monitor his vital signs carefully.

C28 C4 3
A patient with a 10-year history of scleroderma is referred for physical
therapy to improve functional status and endurance. The patient was

478026779.doc
Practice Questions 5

C29 C8 2 B. discontinue the treatment; and discuss the situation with


A patient is referred to physical therapy following a fall injury (fractured the patient and his referring physician.
left hip with ORIF). Medical history reveals a diagnosis of Stage I C. recommend to the patient that he get a second medical
Alzheimer’s disease. At this stage, the behaviors the therapist would opinion as the surgery seems unsuccessful.
NOT expect to find are: D. re-examine the patient and implement a modified therapy
A. anxiety and irritability. program.
B. fragmented memory.
C. language comprehension problems. C35 C1 2
D. restlessness and sundowning. During a postural screen for chronic shoulder pain in a recreational
swimmer, the therapist observes excessive internal rotation of the
C30 C6 2 shoulders and winging of the scapula during overhead motion.
A patient recovering from stroke with minimal lower extremity Intervention should focus on:
weakness and spasticity is able to walk without an assistive device. A. strengthening middle and lower trapezius and stretching
The therapist observes that as he walks he hikes his pelvis on the of pectoral muscles.
affected side during the swing phase. The BEST initial intervention is: B. strengthening of pectoral muscles and stretching of
A. bridging exercises progressing to sit-to-stand training. upper trapezius.
B. marching while sitting on a therapy ball. C. strengthening of rhomboids and stretching of upper
C. partial wall squats using a small ball held between the trapezius.
knees.. D. strengthening of upper trapezius and stretching of
D. standing, marching with manual pressure applied pectoral muscles.
downward on the pelvis. C36 C4 2
Re-examination of a patient with a dermal ulcer over the coccyx
C31 C7 3 reveals a wound exposing the deep fascia. There is no necrotic tissue,
The therapist is instructing a patient with traumatic brain injury how to exudate is minimal, and the borders of the ulcer are diffusely covered
lock the brakes on his wheelchair. He is right-handed and his right with granulation tissue Previous treatment has included daily whirlpool
upper extremity is more affected than his left. To obtain optimal and wet-to-dry dressings with normal saline. Based on the re-
results, the BEST training strategy is to: examination, intervention should consist of:
A. guide his right hand through the locking motions, then A. calcium alginate dressings
his left. B. continuation of the same treatments.
B. have him practice brake locking using his left hand to C. whirlpool and hydrogel dressings
assist his right. D. wound irrigation with pressures below 15 psi.
C. have him practice locking the brakes first with his left
hand, and then his right. C37 C1 2
D. verbally talk him through the locking motions practicing The spinal defect shown in the diagram should be managed with
with both hands simultaneously. avoidance of lumbar spinal:

C32 C7 3
A physical therapist wants to examine the effects of PNF using the
technique of contract-relax on shoulder ROM. A group of 10 patients
with adhesive capsulitis were recruited. A matched group of patients
were given straight plane active-assisted exercise for the same length
of time (3 times/week for 6 weeks). In this study the independent
variable is:
A. active-assisted exercise.
B. adhesive capsulitis.
C. PNF contract-relax technique.
D. ROM.

C33 C3 1 A. extension.
A therapist is examining a patient with a 12-year history of B. flexion.
emphysema. The clinical finding the therapist would NOT expect to C. lateral flexion.
find is: D. rotation.
A. clubbing.
B. cor pulmonale. C38 C2 2
C. cyanosis. A computer specialist is unable to work because of weakness and
D. decreased A-P to lateral chest ratio. altered sensation in her dominant right hand. She complains of pain
and tingling of the thumb, index finger, long finger, and radial half of
C34 C1 3 the ring finger. The therapist observes thenar weakness and atrophy.
A patient is receiving physical therapy following an ACL repair. After 4 Strength, reflexes, and sensation are within normal limits throughout
weeks of treatment the patient still complains of pain and instability the remainder of the right upper extremity. Her signs and symptoms
even though he reports he has been faithful with his home exercise are characteristic of:
program and wearing his orthosis. The original referral was for 7 to 8 A. carpal tunnel syndrome.
weeks of physical therapy. The therapist’s BEST course of action is to: B. cervical root compression.
A. complete the full 8 weeks of treatment and carefully C. pronator teres syndrome.
document his lack of improvement to ensure full D. ulnar nerve compression.
insurance coverage.
C39 C2 2

478026779.doc
6 Practice Questions

A patient presents with rapidly progressive symmetrical weakness that


started in the distal lower extremity muscles but now has ascended to
include proximal trunk and upper extremity muscles. The motor
segments of the lower cranial nerves are also showing impairment.
The patient complains of abnormal sensations of tingling and burning
of the affected extremities. Consciousness, cognition, and
communication are all normal. These signs and symptoms are
characteristic of:
A. amyotrophic lateral sclerosis.
B. Guillain-Barré syndrome.
C. multiple sclerosis.
D. post-polio syndrome.

C40 C3 3
A patient recovering from surgery for triple coronary artery bypass
grafts is scheduled to begin a Phase III cardiac rehabilitation program.
During the resistance training portion of the circuit training program,
the therapist instructs the patient to AVOID the Valsalva maneuver
because:
A. a cholinergic or vagal response can occur.
B. heart rate and blood pressure are likely to be elevated.
C. slowing of pulse and increased venous pressure are
possible.
D. the decreased return of blood to the heart can lead to
pitting edema.

478026779.doc
Practice Questions 7

C41 C4 2 B. posterior tibialis and intrinsic foot muscles.


A patient experiences color changes in the skin during position C. quadriceps femoris and anterior tibialis.
changes of the foot. During elevation, pallor develops. When the limb D. soleus and gastrocnemius.
is then positioned in the seated hanging position, hyperemia develops.
These changes are indicative of:
A. arterial insufficiency.
B. chronic venous insufficiency.
C. deep vein thrombophlebitis.
D. lymphedema.

C42 C8 2
A 73 year-old is referred to physical therapy for an examination of
balance. He has a recent history of falls (two in the last 6 months).
Based on knowledge of balance changes in the elderly and scoring of
standardized balance measures, the test data that BEST indicates
increased fall risk is:
A. Berg Balance score of 50.
B. Functional Reach of 7 inches.
C. Timed Get Up & Go test result of 13 seconds.
D. Tinetti Performance Oriented Mobility Assessment
(POMA) score of 27.

C43 C6 3
A patient walks with a Trendelenburg gait. The MOST appropriate
intervention to correct this problem is:
A. bridging, holding with Theraband around both thighs.
B. half kneeling, weight shifting onto the weak side (foot).
C. standing, stepping with the weaker limb, forward and
backward.
D. supine, lateral leg slides.

C44 C3 2
A 72 year-old patient is walking on a treadmill in the physical therapy
department while his vital signs are being monitored. It is noted that
his SaO2 drops from 97% to 95%. In this case, it would be BEST to:
A. not use supplemental O2.
B. place 2 liters of O2 by nasal cannula on the patient for
the remainder of the exercise session.
C. place a 100% O2 face mask on the patient for the
remainder of the exercise session.
D. place a 40% O2 face mask on the patient for the
remainder of the exercise session.

C45 C8 2
The therapist is evaluating the needs of a 6 year-old child who is
diagnosed with myelodysplasia at the T10 level. The therapist
determines the most appropriate mobility device for this child to use in
the school environment is a:
A. bilateral HKAFO.
B. bilateral KAFO.
C. lightweight wheelchair.
D. parapodium.

C46 C1 3
A patient has limited right rotation caused by left thoracic facet joint
capsular tightness. The intervention that would best facilitate improved
right rotation in sitting is:
A. trunk extension with left rotation.
B. trunk extension with right rotation.
C. trunk flexion with left rotation.
D. trunk flexion with right rotation.

C47 C1 1
EMG activity in the lower extremities during erect standing is
continuous in the:
A. anterior tibialis and peroneals.

478026779.doc
8 Practice Questions

C48 C7 3 B. begin with an interval walking program, exercising only to


A patient falls while walking in the parallel bars. The therapist is the point of pain.
required to fill out an incident report of the event. Information in the C. utilize a walking program of moderate intensity,
report should include the names of those involved and: instructing the patient that some pain is expected and to
A. a description of the event and the corrective actions that be tolerated.
will be taken. D. utilize nonweightbearing exercises such as cycle
B. the date of the occurrence, a description of the event, and ergometry.
the cause of the fall.
C. what occurred, when and where it occurred, and witness
statements.
D. witness reports and therapist’s opinion as to the cause.

C49 C5 3
Four days ago, a patient sustained a deep contusion of the right
lateral thigh as a result of a blow on the leg by a steel beam. Following
several cryotherapy treatments, the therapist decides to apply
ultrasound. The parameters that are most appropriate in this case are:
A. continuous US at 1 MHz.
B. continuous US at 3 MHz.
C. pulsed US at 1 MHz.
D. pulsed US at 3 MHz.

C50 C2 2
A 99 year-old woman was found unconscious at home. Two days later
the therapist examines her in the hospital. Findings include normal
sensation and movement on the right side of the body with impaired
sensation (touch, pressure, proprioception) and paralysis on the left
side of the body. The left side of her lower face and her trunk are
similarly impaired. The MOST LIKELY location of the lesion is the:
A. left parietal lobe.
B. left side of brainstem.
C. right parietal lobe.
D. spinal cord.

C51 C2 1
The loss of sensory function in peripheral neuropathy is often among
the first noticeable symptoms. If more than one nerve is involved, the
sensory loss typically appears as:
A. allodynia of the feet accompanied by pronounced
dorsiflexor weakness.
B. bandlike dysesthesias and paresthesias in the hips and
thighs.
C. paresthesias affecting primarily the proximal limb
segments and trunk.
D. stocking and glove distribution of the lower and upper
extremities.

C52 C3 2
The therapist is supervising a Phase II cardiac rehabilitation class of
10 patients. One of the patients, who is being monitored with
radiotelemetry, is having difficulty. The therapist decides to terminate
the patient’s exercise session and reduce the exercise intensity upon
observing:
A. 1 mm ST-segment depression, upsloping.
B. a 2° AV heart block.
C. an increase in HR 20 BPM above resting.
D. an increase in systolic BP to 150 and diastolic BP to 90.

C53 C4 3
A patient presents with severe claudication which is evident when he
walks distances greater than 200 feet. He also exhibits muscle fatigue
and cramping of both calf muscles. Upon examination, the therapist
finds his skin is pale and shiny with some trophic nail changes. The
BEST choice for intervention is to:
A. avoid any exercise stress until he has been on calcium
channel blockers for at least 2 weeks.

478026779.doc
Practice Questions 9

C54 C8 3 D. unilateral P/A pressure at a 60 degree angle on the left


A 72 year-old is hospitalized with diabetes and a large stage II plantar transverse process of T6 while stabilizing T5.
ulcer located over his right heel. He has been non-weightbearing for
the past 2 weeks as a result of the ulcer. The BEST intervention is: C60 C2 2
A. clean and bandage with a sterile gauze dressing A patient presents with an acute onset of vertigo over night.
B. clean and debride the wound, and apply a hydrogel Symptoms worsen with rapid change in head position. If the head is
dressing. held still, symptoms subside usually within 1 to 2 minutes. The MOST
C. request a surgical consult as available wound dressings likely cause of this patient’s problem is:
will not promote healing. A. acoustic neuroma.
D. wash the foot and apply skin lubricants followed by a B. benign paroxysmal positional vertigo.
transparent film dressing. C. bilateral vestibular neuritis.
D. Ménière’s disease.
C55 C6 3
The therapist is prescribing a wheelchair for a patient with left
hemiplegia. The MOST appropriate feature to include in this
prescription is:
A. a 17.5 inch seat height.
B. a 20 inch seat height.
C. detachable arm rests
D. elevating legrests.

C56 C3 3
A patient who is participating in a cardiac rehab program suddenly
collapses and falls to the floor. The physical therapist is the lone
rescuer on site. The therapist checks for a pulse and finds the patient
unresponsive. The therapist calls the emergency response number,
and then opens the airway, checks breathing. The patient is not
breathing, The therapist should:
A. give 2 rescue breaths followed by 15 chest compressions,
repeating the cycle.
B. give 2 rescue breaths followed by 30 chest compressions,
repeating the cycle.
C. give chest compressions only at a rate of 100 per minute.
D. use the AED to shock the patient immediately; if
unsuccessful then initiate CPR.

C57 C1 2
A patient who was casted for 3 weeks following a Grade III right ankle
sprain has been referred to physical therapy for mobility exercises.
Examination shows a loss of 10 degrees of dorsiflexion. The patient
will have the MOST difficulty in:
A. ambulating barefoot.
B. ambulating over rough surfaces.
C. descending a ramp.
D. descending stairs.

C58 C6 3
The torque output produced in the sitting position during isokinetic
exercise involving the hamstrings is:
A. higher due to eccentric assistance of the quadriceps.
B. higher than the torque actually generated by the
contracting hamstrings.
C. lower due to resistance of the quadriceps.
D. lower than the torque actually generated by the
hamstrings.

C59 C1 3
A manual therapy technique utilized to correct a closing restriction of
T5 on T6 is:
A. central P/A pressure at a 45 degree angle on the spinous
process of T5 while stabilizing T6.
B. central P/A pressure at a 60 degree angle on the spinous
process of T6 while stabilizing T5.
C. unilateral P/A pressure at a 45 degree angle on the right
transverse process of T6 while stabilizing T5.

478026779.doc
10 Practice Questions

C61 C3 2 shows medial femoral torsion and toeing-in position of the feet. The
A patient with a significant history for coronary artery disease tells the lower extremity position may be indicative of excessive hip:
therapist he is currently taking atropine with careful monitoring. Based A. anteversion.
on knowledge of the effects of this medication the therapist expects: B. lateral/external rotation.
A. bradycardia at rest and with exercise. C. medial/internal rotation.
B. increased heart rate and contractility at rest. D. retroversion.
C. reduced blood pressure at rest and with exercise.
D. reduced myocardial ischemia and heart rate.

C62 C4 2
A 67 year-old patient recovering from stroke is on warfarin
(Coumadin). During his rehabilitation, it would be important to watch
for:
A. cellulitis and xeroderma.
B. edema and dermatitis.
C. hematuria and ecchymosis.
D. palpitations and edema.

C63 C8 3
A 91 year-old patient has reduced vision as a result of bilateral
cataracts. Which of the following is NOT an appropriate intervention
for this patient?
A. avoiding having him walk on shiny floor surfaces.
B. highlighting steps with pastel colors of blues and greens.
C. minimizing visual distractions in his immediate
environment.
D. using high illumination for reading and ADLs.

C64 C6 2
A patient is having difficulty with stair climbing. During ascent, he is
able to position his foot on the step above but is unable to transfer the
weight of his body up to the next stair level. The BEST intervention to
solve this problem is:
A. hooklying, bridging.
B. plantigrade, knee flexion with hip extension.
C. standing, partial wall squats.
D. standing, side steps.

C65 C4 2
A 72 year-old patient has been hospitalized, on complete bedrest, for
10 days. A physical therapy referral requests mobilization out-of-bed
and ambulation. The patient complains that today his right calf is
aching. If he gets up and moves around he is sure he will feel better.
The therapist’s examination reveals calf tenderness with slight
swelling and warmth. The therapist decides to:
A. ambulate the patient with his support stockings on.
B. begin with ankle pump exercises in bed and then
ambulate.
C. postpone ambulation and report the findings immediately.
D. use only AROM exercises with the patient sitting at the
edge of the bed.

C66 C8 1
A nine year-old boy with Duchenne’s muscular dystrophy is referred
for homecare. The physical therapist should BEGIN the examination
by:
A. asking the child and his parents to describe the boy’s
most serious functional limitations.
B. asking the parents to outline the boy’s past rehabilitation
successes.
C. performing a complete motor examination.
D. performing a functional examination using the weeFIM.

C67 C1 1
During an examination of an adolescent female who complains of
anterior knee pain, the therapist observes that the lower extremity

478026779.doc
Practice Questions 11

C68 C1 3
A patient with osteoporosis and no fractures complains of increased C74 C3 2
mid and low back pain during breathing and other functional activities. A patient is referred for physical therapy following an exercise
The MOST appropriate interventions for this patient include patient tolerance test. The physician reports the test was positive and had to
education and: be terminated at 7 minutes. Based on the therapist’s knowledge of this
A. trunk extension and abdominal stabilization exercises. procedure, the therapist expects the patient may have exhibited:
B. trunk flexion and extension exercises. A. a hypertensive response with a BP of at least 170/95.
C. trunk flexion, and rotation exercises. B. ECG changes from baseline of 1 mm ST-segment
D. trunk rotation and abdominal stabilization exercises. elevation.
C. increasing angina and dyspnea with progressive
C69 C1 2 increases in the treadmill speed and grade.
A single, 22 year-old woman, who is 3 months pregnant, arrives at a D. ST segment depression from baseline of 3 mm horizontal
therapist’s private practice complaining of shoulder and leg pain. She or downsloping depression.
has a black eye and some bruising at the wrists. The state in which C75 C1 2
the therapist practices has direct access. An appropriate course of A college soccer player sustained a hyperextension knee injury when
action is: kicking the ball with his other lower extremity. The patient was taken to
A. administer massage for bruising, TENs and ice modalities the emergency room of a local hospital and was diagnosed with "knee
for pain, as indicated by the examination findings. sprain". He was sent to physical therapy the next day for aggressive
B. direct the patient to the nearest Ambulatory Care Center rehabilitation. As part of the examination to determine the type of
for physician evaluation. treatment plan to implement, the therapist conducts the test shown in
C. do a comprehensive examination, and if the therapist the figure. The type of exercise that is indicated in the acute phase of
suspects abuse report the findings to the appropriate treatment if a positive test is found includes:
authorities.
D. perform a comprehensive examination and send her to
the emergency room along with a copy of the findings.

C70 C5 3
A 73 year-old patient presents with a stage III decubitus ulcer on the
plantar surface of the right foot. After a series of conservative
interventions with limited success, the therapist chooses to apply
electrical stimulation for tissue repair. The electrical current BEST
suited in this case is:
A. agility exercises.
A. high volt monophasic pulsed current.
B. closed chain terminal knee extension exercises.
B. low volt biphasic pulsed current.
C. open-chain terminal knee extension exercises.
C. medium frequency beat current.
D. plyometric functional exercises.
D. medium frequency burst current.
C76 C4 3
C71 C5 3
A patient with a grade III diabetic ulcer is being treated with a calcium
A patient presents with pain radiating down the posterior hip and thigh
alginate wound dressing. This type of dressing can be expected to:
as a result of a herniated disk in the lumbar spine. The therapist
A. absorb exudate and allow rapid moisture evaporation.
decides to apply mechanical traction. If the patient can tolerate it, the
B. facilitate autolytic debridement and absorb exudate.
PREFERRED patient position is:
C. provide semirigid support for the limb while maintaining a
A. prone with no pillow.
sterile field.
B. prone with pillow under the abdomen.
D. restrict bacteria from the wound while supporting the
C. supine with both knees flexed.
tissues.
D. supine with one knee flexed.
C77 C4 3
C72 C2 3
The therapist receives a referral to treat a patient with a 5-year history
A patient recovering from stroke demonstrates hemiparesis of his right
of cirrhosis and Hepatitis B. The therapist should:
upper extremity and moderate flexion and extension synergies (flexion
A. ask the patient to wear gloves and avoid contact.
stronger than extension). The therapist’s goal is strengthen the
B. avoid direct exposure to blood and body fluids.
shoulder muscles first, specifically the deltoid. The BEST choice is to
C. use contact precautions.
promote:
D. use droplet transmission precautions.
A. abduction with elbow extension.
B. abduction with elbow flexion.
C78 C6 2
C. horizontal adduction with elbow extension.
A patient with a transtibial amputation is learning to walk using a PTB
D. horizontal adduction with elbow flexion.
prosthesis. He is having difficulty maintaining prosthetic stability from
heel-strike to footflat. The muscles that are MOST LIKELY weak are
C73 C2 2
the:
A patient recovering from a middle cerebral artery stroke presents with
A. back extensors.
gaze deviation of the eyes. In this type of stroke the involved eye may
B. hip flexors.
deviate toward:
C. knee extensors.
A. down and out.
D. knee flexors.
B. the hemiplegic side.
C. the sound side.
C79 C3 2
D. up and in.

478026779.doc
12 Practice Questions

A patient with a history of coronary artery disease and recent A. recommend the mother feed the child baby food instead of
myocardial infarction is exercising in an inpatient Cardiac Rehab cereal for a few more months.
Program. As the patient is new, continuous ECG telemetry monitoring B. recommend the mother return to breast feeding for a few
is being done. The therapist observes the following. The BEST course more months.
of action is to: C. reposition the child in a proper sitting position using
postural supports.
D. work on desensitizing the gag reflex.

C84 C1 3
A patient is sent to physical therapy with a diagnosis of "frozen
shoulder". The MOST effective mobilization technique for restricted
shoulder abduction is:
A. inferior glide at 55 degrees of abduction.
B. inferior glide at 95 degrees of abduction.
C. lateral glide in neutral position.
A. activate the emergency medical response team. D. posterior glide at 10 degrees of abduction.
B. have the patient sit down and sent back to the room after
a brief rest period.. C85 C1 2
C. have the patient sit down, continue monitoring, and notify A patient presents with insidious onset of pain in the jaw that is
the physician immediately. referred to the head and neck regions. As best as he can recall, it may
D. have the patient sit down, rest, then resume the exercise be related to biting into something hard. Cervical ROM is limited in
at a lower intensity. flexion by 20 degrees, cervical lateral flexion limited to the left by 10
C80 C7 3 degrees. Mandibular depression is 10mm with deviation to the left,
A patient recovering from traumatic brain injury is unable to bring the protrusion is 4mm, and lateral deviation is 15mm to right and 6mm to
right foot up on the step during stair climbing training. The BEST left. Based on these findings the diagnosis for this patient would be:
training activity is to: A. capsule-ligamentous pattern of TMJ on the left.
A. passively bring the foot up and place it on the 7 inch step. B. cervical spine and TMJ capsular restrictions on the left.
B. practice marching in place. C. weak lateral pterygoids on the left.
C. practice stair climbing inside the parallel bars using a 3 D. weak lateral pterygoids on the right.
inch step.
D. strengthen the hip flexors using an isokinetic training C86 C7 3
device before attempting stair climbing. A 35 year-old administrative assistant and mother of three is being
treated for a Colles’ fracture. Her husband wants to look at her
C81 C3 3 medical record. The physical therapist should:
Which of the following interventions is NOT beneficial for a patient with A. deny access to the chart unless written permission by his
a right lower lobe viral pneumonia: wife is granted.
A. ambulation activity, monitoring SaO2, HR, RR, and BP. B. give him the chart as he is a spouse and has a right to
B. breathing exercises, encouraging right lateral costal view the information.
expansion. C. let him look at the chart and be available to answer any
C. postural drainage, percussion, and shaking to the right questions.
lower lobe. D. not let him look at the chart because he may misinterpret
D. teaching an independent exercise program to the patient. the documentation.
C87 C3 2
C82 C4 2 A patient with a recent history of rib fractures suddenly becomes short
A patient suffered a spinal cord injury with a complete injury (ASIA of breath during secretion removal techniques. The patient looks
Level A) at T10. It is now three months post-injury and she is refusing panicked and complains of sharp pain in the left chest. A quick screen
to participate in her functional training program because the major shows a deviated trachea to the right among other signs and
focus is wheelchair independence. She is sure she is going to walk symptoms. The MOST LIKELY explanation for the above is:
again. The therapist’s BEST approach is to: A. angina.
A. discuss the harmful effects of denial and restrict all B. mucous plugging of an airway.
discussions to promoting wheelchair independence. C. pneumothorax.
B. outline realistic short term goals to improve independence D. pulmonary emboli.
while maintaining for the possibility of further recovery.
C. refer the patient for psychological counseling and C88 C5 3
discharge her from P.T. A patient presents with supraspinatus tendinitis. After the initial
D. send the patient home for a short time so she will cryotherapy, the therapist decides to apply ultrasound. To effectively
recognize the need for wheelchair training. treat the supraspinatus tendon, the therapist would place the shoulder
joint in:
C83 C8 3 A. adduction and external rotation.
The therapist is on a home visit, scheduled at lunchtime, visiting an 18 B. adduction and internal rotation.
month-old child with moderate developmental delay. The therapist C. slight abduction and external rotation.
notices the child and mother are experiencing difficulties with feeding. D. slight abduction and internal rotation.
The child is slumped down in the highchair and is unsuccessfully
attempting to use a raking grasp to lift cereal pieces to her mouth. C89 C2 3
Both the child and mother are frustrated. The FIRST intervention
should be to:

478026779.doc
Practice Questions 13

A patient is 5 weeks poststroke and is demonstrating good recovery of


her right upper extremity, characterized as stage 4 recovery stage.
The BEST choice for a training activity is to have the patient:
A. bear weight on the extended right arm in sitting.
B. put on socks and shoes while in sitting.
C. reach forward to bear weight with the right arm extended
against the wall.
D. reach overhead with right arm straight.

C90 C2 2
A patient is 2 days post left CVA and has just been moved from the
intensive care unit to a stroke unit. When beginning the examination,
the therapist finds the patient’s speech slow and hesitant. He is limited
to one and two-word productions and his expressions are awkward
and arduous. However, he demonstrates good comprehension. His
difficulties are consistent with:
A. Broca’s aphasia.
B. dysarthria.
C. global aphasia.
D. Wernicke’s aphasia.

C91 C2 3
A patient recovering from traumatic brain injury is functioning at Stage
IV on the Rancho Los Amigos Levels of Cognitive Functioning Scale.
During the therapist’s initial examination the patient becomes agitated
and tries to bite the therapist. The BEST course of action is to:
A. engage in a calming activity and document the behaviors.
B. postpone the examination for one week and then try
again.
C. postpone the examination until later in the day when the
patient calms down.
D. restructure the formal examination so the therapist can
complete it in three very short sessions.

C92 C3 1
A therapist is working on a cardiac care unit in an acute care facility.
After exercising a patient recovering from a ventricular infarct, the
therapist notices fatigue and dyspnea after mild activity. Later that
day, on a return visit, the therapist notices the patient has a persistent
spasmodic cough while lying in bed. HR is rapid (140) and slight
edema is evident in both ankles. The patient appears anxious and
agitated. The therapist suspects:
A. developing pericarditis.
B. impending myocardial infarction.
C. left ventricular failure.
D. right ventricular failure.

478026779.doc
14 Practice Questions

C93 C3 2 C99 C8 3
A marathon runner is examined in physical therapy for anterior pain in The therapist is treating a child with mild developmental delay
the right lower leg. Her resting heart rate is found to be 46 bpm. The secondary to 7 weeks prematurity at birth. The child is now 8 months
MOST LIKELY explanation for this is that: old and is just learning to sit. The BEST choice for training activity is:
A. a compensatory response to prolonged endurance activity A. prone tilting reactions.
is depressed heart rate with decreased stroke volume. B. sideward protective extension in sitting.
B. a low heart rate is suggestive of a hypotensive disorder. C. standing tilting reactions.
C. incipient coronary pathology is often characterized by an D. supine tilting reactions.
abnormally low heart rate.
D. prolonged endurance training has resulted in a low heart
rate.

C94 C4 2 C100 C3 2
A patient has been taking corticosteroids (hydrocortisone) for A 80 year-old patient with emphysema and a history of hypertension
management of adrenocortical insufficiency. She is referred to performs a 12 minute walking exercise tolerance test. He was able to
physical therapy for mobility training following a prolonged walk 1,106 feet. His vital signs prior to exercise were: HR 104, BP
hospitalization. Potential adverse effects that one can expect from 130/76, SaO2 93%. At peak exercise his vital signs were: HR 137, BP
prolonged use of this medication include: 162/74, SaO2 92%. To calculate his exercise intensity parameters, the
A. atrophy and osteoporosis. BEST method to use is:
B. confusion and depression. A. 40 to 50% of Max METs.
C. decreased appetite and weight loss. B. 40 to 85% HR reserve (Karvonen’s formula).
D. hypotension and myopathy. C. 70 to 80% of HRmax
D. 70 to 85% of age adjusted predicted HRmax
C95 C4 1
A therapist is treating a patient with deep partial-thickness burns over C101 C8 2
35% of the body (chest and arms). Wound cultures reveal a bacterial An expected outcome for a fifteen year-old boy with Duchenne’s
count in excess of 105 per gram of tissue on the anterior left arm The muscular dystrophy is:
therapist can reasonably expect: A. dependent wheelchair mobility.
A. the burn area is pain free as all nerve endings in the B. independent in ambulation with Lofstrand crutches.
dermal tissue were destroyed. C. independent in ambulation with no assistive devices
B. the infected wound can convert the area to a full-thickness D. independent in wheelchair mobility.
burn.
C. the risk of hypertrophic and keloid scars is low as there is C102 C1 1
no viable tissue. When performing scoliosis screening in a school setting, the optimal
D. with antibiotics spontaneous healing can be expected. age to screen for girls is:
A. 15-17.
C96 C6 3 B. 6-8.
Recently, a 10 year-old patient has begun walking with supination of C. 9-11.
her foot. With her shoe off, the therapist finds a new callus on the D. 12-14.
lateral side of the metatarsal head of the 5th toe. The BEST choice for
orthotic prescription is: C103 C1 3
A. scaphoid pad. A patient with a confirmed left C6 nerve root compression due to
B. Thomas heel. foraminal encroachment complains of pain in his left thumb and index
C. viscoelastic shoe insert with forefoot lateral wedge. finger. The MOST effective cervical position to alleviate this radicular
D. viscoelastic shoe insert with forefoot medial wedge. pain in weightbearing is:
A. left sidebending.
C97 C6 3 B. lower cervical extension.
A 74 year-old patient is recovering from a right total hip replacement C. lower cervical flexion.
(posterolateral incision, cementless fixation). The MOST appropriate D. right rotation.
type of bed-to-wheelchair transfer to teach is to have the patient use
a: C104 C1 1
A. lateral slide transfer using a transfer board. A patient is standing with excessive subtalar pronation. Possible
B. squat- pivot transfer to the surgical side. correlated motions or postures are:
C. stand-pivot transfer to the sound side. A. tibial and femoral external rotation, with pelvic external
D. stand-pivot transfer to the surgical side. rotation.
B. tibial and femoral internal rotation with pelvic external
C98 C6 3 rotation.
A patient with paraplegia at the T10 level wants to participate in C. tibial, femoral, and pelvic external rotation.
wheelchair basketball. He asks the therapist what options he should D. tibial, femoral, and pelvic internal rotation.
look for in a wheelchair. The therapist tells him it would be important to
include: C105 C7 3
A. a folding frame. When performing a chart audit, the therapist realizes a date of service
B. a mid-scapular seat back. was documented inappropriately. The therapist should:
C. a rigid frame. A. erase the incorrect date; then make and date the
D. hard-rubber tires. correction.

478026779.doc
Practice Questions 15

B. put a single line through the incorrect date, initial, then


make and date the correction.
C. use "white out", initial; then add the correct date of
service.
D. write over the date with a different color ink, then make
and initial the correction.

C106 C5 3
A therapist is applying high volt pulsed current to the vastus medialis
to improve patellar tracking during knee extension. The patient
complains that the current is uncomfortable. To make the current more
tolerable to the patient, yet maintain a good therapeutic effect, the
therapist should consider adjusting the:
A. current intensity.
B. current polarity.
C. pulse duration.
D. pulse rate.

478026779.doc
16 Practice Questions

C107 C2 2 B. frustration because of an inability to communicate.


A patient has a 10-year history of multiple sclerosis and presents with C. inattention as a result of short term memory loss.
drooping of the right upper eyelid, constriction of the pupil, and D. sundowning behavior
vasodilation with absence of sweating on the face and neck. These
signs are characteristic of: C113 C6 3
A. Argyll Robertson pupil. To increase the stride length of a patient with a right transfemoral
B. homonymous hemianopsia. amputation who uses a total contact prosthesis, the therapist should:
C. Horner’s syndrome. A. facilitate the gluteals with tapping over the muscle belly.
D. nystagmus. B. provide anterior directed resistance to the right PSIS
during swing.
C108 C2 2 C. provide posterior directed resistance to the left ASIS
A patient presents with symptoms of uncoordinated eye movements during swing.
and profound gait and trunk ataxia. He has difficulty with postural D. provide posterior directed resistance to the right ASIS
orientation to vertical and tends to tip over even if his eyes are open. during stance.
Examination of the extremities reveals little change in tone or
coordination. The therapist suspect involvement of the: C114 C6 3
A. basal ganglia. A patient with a complete C7 spinal cord injury is having difficulty with
B. premotor cortex. pushups while in his wheelchair. The MOST appropriate lead-up
C. spinocerebellum. activity to enhance wheelchair pushups is:
D. vestibulocerebellum. A. prone-on-elbow pushups.
B. shoulder shrugs.
C109 C3 1 C. supine bench press using 50% one repetition max.
A patient with a 10-year history of diabetes complains of cramping, D. supine-on-elbows pushups.
pain, and fatigue of the right buttock after walking 400 feet or climbing
stairs. When he stops exercising, the pain goes away immediately. C115 C7 3
The skin of the involved leg is cool and pale. The therapist checks his A therapist is concerned that a student PT, who is on a final clinical
medical record and finds no mention of this problem. The therapist rotation, is having difficulty interacting with the patients. Specifically,
suspects: the student does not seem to be willing to listen or demonstrate
A. peripheral arterial disease. tolerance and sensitivity to patient needs. The MOST appropriate
B. peripheral nerve injury. conclusion the therapist can reach is that affective objectives for the
C. Raynaud’s phenomenon. clinical education experience are not being met. The primary deficit is:
D. spinal root impingement. A. Level 1.0 Receiving.
B. Level 3.0 Valuing.
C110 C4 2 C. Level 4.0 Organization.
A patient with low back pain has marked elevation of blood pressure. D. Level 5.0 Adherence to a professional code of ethics.
He complains of mild to severe midabdominal pain that increases
upon exertion. Palpation reveals a pulsing mass in the lower C116 C7 3
abdomen. The therapist should: A therapist wants to investigate the effectiveness of use of the
A. discontinue treatment and notify his physician therapeutic pool for decreasing pain in a group of patients with
immediately. fibromyalgia. Two groups of patients were recruited. One group was
B. instruct in relaxation exercises as a pulsating mass is not assigned to exercises and walking in the pool 3 times/week for 6
unusual with hypertension. months. The other group was assigned to a gym walking program for
C. instruct the patient to contact his physician at the the same amount of time. At the end of the study, outcomes were
conclusion of therapy. assessed using the McGill Pain Questionnaire and the Health Status
D. provide hot packs to the abdomen to help relieve the Questionnaire. In order to improve reliability, the lead investigator
muscle spasm. should:
A. have another therapist reassess after 6 months and
C111 C4 3 compare to normalized scores.
A patient with a 12-year history of diabetes and a small, purulent B. have the same therapist reassess the patients after 6
tunneling wound located on the left heel is referred for wound lavage. months.
The therapist’s BEST choice is to irrigate the wound using: C. perform all the final assessments himself and compare to
A. a syringe with Dakin’s solution while the patient is in the the initial assessments performed by a core group of
whirlpool. therapists.
B. hydrogen peroxide spray. D. utilize a core of 4 experienced therapists to randomly
C. whirlpool with povidone-iodine. complete all the assessments.
D. whirlpool with water temperature at 20°C .
C117 C3 2
C112 C8 1 An apparently healthy individual has many risk factors for coronary
An 85 year-old wheelchair dependent resident of a community nursing artery disease. He is interested in improving his overall fitness and
home has a diagnosis of organic brain syndrome, Alzheimer’s type, cardiac health. Following an exercise tolerance test, which was
Stage 2. During the therapist’s initial interview, the patient asymptomatic, he is referred for an exercise class. The most accurate
demonstrates limited interaction, mild agitation, and keeps trying to measure of exercise intensity to monitor during his first exercise
wheel her chair down the hall. As it is late in the day, the therapist session is:
decides to resume the examination the next morning. The patient is A. heart rate.
most likely exhibiting: B. MET level.
A. disorientation to time and date. C. rating of perceived exertion (RPE).

478026779.doc
Practice Questions 17

D. respiratory rate. electrical stimulation to stimulate the strengthening of the quadriceps


muscle. The BEST choice of electrode size and placement is:
C118 C8 2 A. large electrodes, closely spaced.
A mother brings her 8 week-old infant to be examined at Early B. large electrodes, widely spaced.
Intervention because she noticed that the infant was taking steps in C. small electrodes, closely spaced.
supported standing at two weeks but was not able to do it now. The D. small electrodes, widely spaced.
therapist should:
A. explain that this is normal and that the stepping was a C124 C2 3
newborn reflex that has gone away. A 65 year-old patient is recovering from a right CVA. Due to a series
B. explain this was due to a stepping reflex that will re- of medical complications she is still bedridden 6 days post-stroke. The
emerge again around 10 months. therapist wants to reduce the expected negative effects of developing
C. recommend that a full developmental exam be performed spasticity. The BEST choice of bed position for this patient is:
by the Early Intervention team. A. sidelying on the affected side, with the affected shoulder
D. recommend that the mother bring the infant to a pediatric positioned directly underneath, the hip slightly extended
neurologist. with knee flexed on a pillow.
B. sidelying on the sound side, affected arm and leg extended
C119 C1 1 at the side, with a pillow between the knees.
A patient complains of pain with mouth opening that makes it difficult C. supine, trunk in midline with small pillow under the scapula,
for her to eat foods that require chewing. Examination revealed active arm extended on supporting pillow, and a small towel roll
mouth opening to be within normal limits of: under the knee.
A. 15-24mm. D. supine, trunk in slight lateral flexion to the sound side with
B. 35-44mm. elbow flexed and supported on a pillow, leg straight.
C. 50-64mm.
D. 65-74mm. C125 C2 1
If the subject’s vision is blocked either by having the subject close the
eyes or by placing a barrier between the part being tested and the
subject’s eyes, the therapist can effectively examine:
A. conscious proprioception but not discriminative touch.
B. discriminative touch and fast pain but not proprioception.
C. somatosensory integrity.
C120 C1 3 D. vestibular/visual/somatosensory integration.
Correction of flexible forefoot varus with excessive subtalar pronation
is accomplished by a customized orthosis with: C126 C3 2
A. lateral forefoot posting. A therapist is examining a patient in the coronary intensive care unit.
B. medial forefoot and rearfoot varus posting. On auscultation the therapist hears an adventitious S3 heart sound.
C. medial forefoot posting. This finding is indicative of:
D. rearfoot varus posting. A. aortic valve dysfunction.
B. congestive heart failure.
C121 C1 2 C. pericarditis.
A patient presents with complaints of tingling and paresthesias in the D. pulmonary valve dysfunction.
median nerve distribution of the right forearm and hand. The following
tests were found negative bilaterally: Adson, hyperabduction,
costoclavicular, Phalen’s, and the ulnar nerve Tinel sign. Based on
this information, the diagnosis that has NOT been ruled out is:
A. carpal tunnel syndrome.
B. pronator teres syndrome.
C. thoracic outlet syndrome.
D. ulnar nerve entrapment.

C122 C7 3
A patient scheduled for a 30-minute treatment session arrives 10
minutes late. The subsequent patients are also scheduled for 30
minutes sessions and there is no break in the therapist’s schedule to
accommodate for the patient’s tardiness. The therapist should:
A. ask the patient if there is a more convenient time for the
appointment in order to complete a 30-minute session.
B. send the patient home until the next scheduled
appointment, with a request to be punctual.
C. treat the patient and bill for 30 minutes of scheduled
treatment time.
D. treat the patient, and bill for the 20-minute session given.

C123 C5 3
A patient has been referred to the therapist s/p fracture of the femur
six months ago. The cast was removed, but the patient is unable to
volitionally contract the quadriceps. The therapist decides to apply

478026779.doc
18 Practice Questions

C127 C3 2
A therapist is asked to advise a healthy 67 year-old individual who
wants to take part in a graduated conditioning program by joining the
"Mall Walkers Club". The therapist’s BEST approach to prescribing the
intensity of exercise for this individual is:
A. 4-8 MET level walking.
B. 70-85% of maximal age-related HR
C. dyspnea scale.
D. HR reserve formula and Ratings of Perceived Exertion.

C128 C4 2
A 72 year-old patient recovering from stroke has been using a bilateral
exerciser (UBE) to strengthen muscles in his affected right upper
extremity. He is now experiencing burning pain in his shoulder that
worsens when his limb is touched or moved. He also presents with
paresthesias and pitting edema in the dorsum of the hand. ROM of the
wrist and fingers is painful and diminished. The therapist’s BEST
course of action is to:
A. discontinue exercise and use ice for pain relief.
B. discontinue UBE exercise, splint the hand and wrist until
pain and swelling disappear.
C. discontinue UBE exercise; use massage and active
assistive ROM.
D. switch to interval exercise and lower the resistance on the
UBE.

C129 C8 2
A 74 year-old patient has had two recent falls coming home from
Bingo after dark. Her outside steps are well lit. She’s unsure why she
has fallen but tells the therapist both times she fell just as she came
into her house, before she even had a chance to put her purse down
and turn on the inside lights. The therapist suspect a problem with:
A. decreased corneal sensitivity.
B. decreased ocular scanning movements.
C. loss of accommodation and near vision.
D. poor light adaptation.

C130 C6 3
A patient presents with 2/5 muscle strength in both lower extremities
and 3/5 strength in the upper extremities. The MOST appropriate
transfer to teach this patient to move from bed to wheelchair is:
A. dependent 1-man squat transfer.
B. sliding board.
C. stand pivot.
D. stand-by assist.

C131 C6 3
A patient with a spinal cord injury at the level of T1 is in the community
phase of his mobility training. In order for him to navigate a standard
height curb with his wheelchair, the therapist tell him to:
A. ascend backwards with the large wheels first.
B. descend backward with the trunk upright and arms hooked
around the push handles.
C. lift the front casters and ascend in a wheelie position.
D. place the front casters down first during descent.

C132 C3 2
A patient with a long history of systemic steroid use for asthma control
has a contraindication for percussion if there is evidence of:
A. barrel chest.
B. BP > 140/90.
C. decreased bone density.
D. intercostals muscle wasting.

478026779.doc
Practice Questions 19

C133 C1 2 D. UCBL insert.


A patient presents with a rapid onset of severe weakness of all small
muscles of the hands, sharp pleuritic pain in the shoulder and
subscapular area, and a hoarse voice for the past three weeks. She is
a hair stylist, has a smoking history of 20 years, is not on any
medications, and has not been ill. Her referral states examine and
treat. Based on the above information this patient is MOST LIKELY
exhibiting symptoms of:
A. a Pancoast tumor.
B. thoracic outlet syndrome
C. ascending Guillain-Barré syndrome.
D. a C5-6 bilateral foraminal stenosis.

C134 C1 2
An examination of a patient reveals the following shoulder signs and
symptoms: excessive AROM and PROM; pain with activity, and on
palpation; normal resisted isometric contractions; a positive load/shift
test; and negative X-ray findings. The MOST LIKELY diagnosis is:
A. atraumatic shoulder instability.
B. impingement.
C. rotator cuff lesion.
D. traumatic anterior shoulder dislocation.

C135 C5 3
The therapist is applying cervical traction using a cervical harness.
The patient complains of pain in the temporomandibular joint during
the treatment. The therapist should consider:
A. decreasing the treatment time.
B. discontinuing traction.
C. readjusting the harness and continuing with the treatment.
D. reducing the traction poundage and continuing with the
treatment.

C136 C6 2
A therapist is treating a patient with a diagnosis of right shoulder
rotator cuff tendonitis. The findings of a worksite ergonomic
assessment indicate that the worker is required to perform repetitive
reaching activities above shoulder height. The most appropriate
worksite modification would be to:
A. allow the worker to take more frequent rests to avoid
overuse.
B. provide the worker with a standing desk for daily activities.
C. provide the worker with a taller, sit-stand chair.
D. reposition the height of the shelf and items to below
shoulder height.

C137 C2 3
A patient has been diagnosed with impingement syndrome of the
shoulder. Following a course of modalities to control pain and
inflammation, progression is to an exercise program to restore normal
function of the shoulder. The BEST PNF diagonal pattern to improve
function of the shoulder is:
A. D1 extension.
B. D1 flexion.
C. D2 extension.
D. D2 flexion.

C138 C6 2
A patient presents with pain of the right Achilles tendon as well as on
the plantar aspect of the right heel. Pain developed insidiously and
has now lasted several months. On gait analysis the therapist
observes abnormal supination throughout the stance phase of gait.
The BEST choice for orthotic intervention is a:
A. cushion heel with a rearfoot valgus post.
B. flexible shoe insert with forefoot varus post.
C. metatarsal pad.

478026779.doc
20 Practice Questions

C139 C3 2 outlined for him to promote independent functional mobility. A review


A patient has a 10 year history of peripheral vascular disease affecting of his history reveals that previously he was the director of his own
the right lower extremity. During auscultation of the popliteal artery the company, with a staff of 20. The MOST appropriate strategy the
therapist would NOT expect to find: therapist can adopt is to:
A. 1+ pulses. A. carefully structure the activities and slow down the pace of
B. 2+ pulses. training.
C. a bruit. B. have him work with a supervisor since he works best with
D. absence of detectable blood flow. people in authority.
C. involve him in goal setting and have him participate in
C140 C4 2 structuring the training session.
A therapist is working in a major medical center and is new to the D. refer him to another therapist who is male in the hopes that
acute care setting. An orientation session for new employees he will have better luck in engaging the patient.
concerns infection control. The therapist learns that the most common
infection transmitted to healthcare workers is:
A. hepatitis A.
B. hepatitis B.
C. HIV.
D. tuberculosis.

C141 C4 2
A patient presents with pain, joint swelling, subcutaneous olecranon
nodules, and increased erythrocyte sedimentation rate. These findings
are characteristic of:
A. fibromyalgia.
B. osteoarthritis.
C. rheumatoid arthritis.
D. systemic lupus erythematosus.

C142 C6 1
A patient with a transfemoral amputation and an above-knee
prosthesis demonstrates knee instability while standing. His knee
buckles easily when he shifts his weight. The therapist suspect the
cause of his problem is a:
A. prosthetic knee set too far anterior to the TKA line.
B. prosthetic knee set too far posterior to the TKA line.
C. tight extension aid.
D. weak gluteus medius.

C143 C6 3
A 17 year-old individual with developmental disabilities is referred to a
wheelchair clinic for a new wheelchair. She presents with a severe
kyphoscoliosis. The therapist determines the BEST wheelchair
modification to order is a:
A. contoured foam seat.
B. firm seat back with lateral posture supports and increased
seat depth.
C. firm seat with lateral knee positioners.
D. sling seat with dense foam cushion.

C144 C6 3
A patient is referred for orthotic gait training after receiving a
reciprocating gait orthosis. In order for this patient to walk correctly, it
is important to instruct her in the correct sequence. She should shift
her weight:
A. onto her crutches and swing both legs through together to
a position in front of her crutches.
B. onto her crutches and swing one leg, then the other
forward.
C. onto her walker and one leg, tuck her pelvis by extending
the upper trunk, and swing her other leg through.
D. onto her walker, extend the upper trunk, and swing both
legs forward together to approach the walker.

C145 C7 3
A patient who is undergoing spinal cord rehabilitation is viewed as
uncooperative by staff. He refuses to complete the training activities

478026779.doc
Practice Questions 21

C146 C7 3 B. pronator teres syndrome.


A group of 10 patients is recruited into a study investigating the effects C. thoracic outlet syndrome.
of relaxation training on blood pressure. One group of patients is D. ulnar nerve entrapment.
scheduled to participate in a cardiac rehabilitation program which
includes relaxation training 3 times a week for 12 weeks. The other
group of patients is instructed to perform activities as usual. At the
conclusion of the study there was no significant difference between
the groups; BP decreased significantly in both groups. The
investigator can reasonably conclude:
A. both groups had blood pressures initially so high that
reductions should have been expected.
B. cardiac rehabilitation is not effective in reducing blood
pressure.
C. the activities of the non-rehab group were not properly
monitored and may account for these results.
D. the rehab group was not properly monitored.

C147 C6 2
A patient is using a right KAFO. During orthotic checkout, the therapist
discover the height of the medial upright is excessive. As she transfers
weight to the orthotic leg during gait, the therapist expects that this
patient will demonstrate:
A. anterior trunk bending.
B. lateral lean toward the left.
C. lateral lean toward the right.
D. posterior trunk bending.

C148 C6 2
During gait, a patient with hemiparesis drags his toes during swing.
Upon further examination, he has weak dorsiflexors (able to lift the
foot against gravity through 1/2 range) and a grade of 2 upon
examining tone in his plantar flexors using the Modified Ashworth
Scale. An appropriate orthotic modification to correct this problem is:
A. a dorsiflexion assist.
B. a dorsiflexion stop.
C. a solid ankle AFO.
D. spiral AFO.

C149 C8 2
A therapist is examining a 24 month-old child and observes that the
child can sit independently, creep in quadruped, pull-to-stand, cruise
sideways, but not walk without support. The therapist concludes that
this child is exhibiting:
A. delay in achieving developmental milestones.
B. normal cephalocaudal motor development.
C. normal gross motor development.
D. slow maturation that is within normal limits.

C150 C1 3
Therapist hand/finger placements for posterior to anterior (PA)
mobilization techniques to improve down-gliding/closure of the T7-8
facet joints should be located at the:
A. spinous process of T6.
B. spinous process of T8.
C. transverse processes of T7.
D. transverse processes of T8.

C151 C1 2
A patient complains of waking up several times at night from severe
"pins and needles" in both hands. On awakening, her hands feel numb
for half an hour, and she complains of clumsiness with fine hand
movements. The therapist’s examination revealed paresthesias in the
medial forearm and hypothenar region; reduced grip and pinch
strength; and normal tendon reflexes. Based on the above
examination findings the MOST appropriate diagnosis is:
A. carpal tunnel syndrome.

478026779.doc
22 Practice Questions

C152 C5 2 A patient is hospitalized in the ICU with extensive trauma following a


An auto mechanic is referred for physical therapy with a diagnosis of motor vehicle accident. A review of her medical record reveals the
degenerative joint disease affecting C2 and C3. The patient complains following lab values: hematocrit 28%, hemoglobin 10 g/100ml, and
of pain and stiffness in the cervical region and transient dizziness with serum WBC 12,000/mm3. The MOST appropriate conclusion the
some cervical motions. The MOST appropriate INITIAL examination therapist can reach is:
procedure is: A. all values are abnormal.
A. a vertebral artery test. B. hematocrit and hemoglobin values are abnormal; WBC is
B. Adson’s maneuver. normal.
C. Lhermitte’s test. C. only hematocrit values are abnormal.
D. Oppenheim test. D. only serum WBC is abnormal.

C153 C2 3 C159 C4 2
A 22 year-old otherwise healthy patient is recovering from a complete A patient with a transfemoral amputation is unable to wear his total
spinal cord injury at the level of L2. Functional expectations for this contact prosthesis for the past 4 days. Examination of the residual
patient include: limb reveals erythema and edema extending over most of the lower
A. ambulation using bilateral AFOs and canes anterior limb. He tells the therapist his limb is very itchy and painful
B. ambulation using bilateral KAFOs and a reciprocating after he scratches it. The MOST LIKELY cause of his symptoms is:
walker. A. cellulitis.
C. ambulation using bilateral KAFOs, crutches and a swing- B. dermatitis.
through gait. C. herpes zoster.
D. wheelchair locomotion using an active duty lightweight D. impetigo.
chair.
C160 C4 3
C154 C2 1 A patient is largely confined to bed and has a stage IV sacral pressure
To examine a patient with a suspected deficit in graphesthesia, the ulcer of three months duration. The BEST choice of intervention is:
therapist would ask the patient, with eyes shut, to identify: A. a two-inch, convoluted foam mattress.
A. a series of letters traced on the hand. B. gentle wound cleansing and wet-to-dry gauze dressings.
B. different objects placed in the hand and manipulated. C. nutritional supplements and pressure relief with a flotation
C. different weighted, identically shaped cylinders placed in mattress.
the hand. D. surgical repair.
D. the vibrations of a tuning fork when placed on a bony
prominence. C161 C6 2
Following a hip fracture that is now healed, a patient presents with
C155 C2 3 weak hip flexors (2/5). All other muscles are within functional limits.
A patient recovering from stroke is ambulatory without an assistive During gait, the therapist expects that the patient may walk with:
device. He demonstrates a consistent problem with an elevated and A. a circumducted gait..
retracted pelvis on the affected side. The BEST therapeutic exercise B. backward trunk lean.
strategy is to manually apply: C. excessive hip flexion.
A. anterior directed pressure during swing. D. forward trunk lean.
B. downward compression during stance.
C. light resistance to forward pelvic rotation during swing. C162 C6 3
D. light resistance to posterior pelvic elevation during swing. A 72 year-old patient with a left transfemoral amputation complains
that his left foot is cramping and when he sits it feels all twisted under
C156 C2 2 him. The therapist’s BEST choice of intervention is:
A therapist suspects lower brainstem involvement in a patient with A. appropriate bed positioning with the residual limb in
amyotrophic lateral sclerosis. Examination findings reveal motor extension.
impairments of the tongue with ipsilateral wasting and deviation on B. hot packs and continuous ultrasound to the residual limb.
protrusion. These findings confirm involvement of cranial nerve: C. icing and massage to the residual limb.
A. IX. D. iontophoresis to the distal residual limb using
B. X. hyaluronidase.
C. XI.
D. XII. C163 C7 3
A researcher uses a group of volunteers (healthy, college students) to
C157 C3 2 study the effects of Swiss ball exercises on ankle ROM and balance
A patient is in the intensive care unit following myocardial infarction. scores. Twenty volunteers participated in the 20 minute ball exercise
Upon examination of the ECG tracings in the medical record the class 3 times a week for 6 weeks. Measurements were taken at the
therapist observes the following changes: ST elevation and T wave beginning and end of the sessions. Significant differences were found
inversion in leads II, III, and AVF. The probable location of the infarct in both sets of scores and reported at the local PT meeting. Based on
is the: this research design, the therapist concludes:
A. inferior wall. A. Swiss ball exercises are an effective intervention to
B. lateral wall. improve ankle stability following chronic ankle sprain.
C. posterior wall. B. the Hawthorne effect may have influenced the outcomes of
D. anterior wall. the study.
C. the reliability of the study was threatened with the
C158 C4 1 introduction of systematic error of measurement.

478026779.doc
Practice Questions 23

D. the validity of the study was threatened with the B. spinocerebellum.


introduction of sampling bias. C. vermis.
D. vestibulocerebellum.
C164 C3 3
A patient has a very large right-sided bacterial pneumonia. Her C170 C3 2
oxygen level is dangerously low. The body position that would MOST A patient is exercising in a Phase 3 outpatient cardiac rehabilitation
LIKELY improve her PaO2 is: program that utilizes circuit training. One of the stations utilizes
A. left sidelying with the head of the bed in the flat position. weights. The patient lifts a 5 lb weight, holds it for 20 seconds and
B. prone-lying with the head of the bed in the Trendelenburg then lowers it slowly. The therapist corrects the activity and tells the
position. patient to reduce the length of the static hold. The static exercise can
C. right sidelying with the head of the bed in the flat position. be expected to produce:
D. supine-lying with the head of the bed in the Trendelenburg A. abnormal oxygen uptake.
position. B. higher heart rate and arterial blood pressure.
C. lower heart rate and arterial blood pressure.
C165 C1 1 D. reduced normal venous return to the heart and elevated
An examination of a 46 year-old woman reveals drooping of the blood pressure.
shoulder, rotatory winging of the scapula, an inability to shrug the
shoulder, and complaints of aching in the shoulder. Based on these C171 C4 3
findings, the cause of these symptoms would MOST LIKELY be due A ten year-old boy with hemophilia fell and injured himself while
to: skateboarding. He was admitted to a pediatric acute care facility and a
A. a lesion of spinal accessory nerve. therapist sees him that afternoon. Examination reveals a hemarthrosis
B. a lesion of the long thoracic nerve. in his left knee. The BEST initial intervention for this patient is:
C. muscle imbalance. A. a hot pack for the knee and instruction in nonweightbearing
D. strain of the serratus anterior. exercises.
B. a pool program to maintain ROM and strength while he is
C166 C5 3 nonweightbearing.
A patient with spastic hemiplegia is referred to the therapist for C. ice, elevation and a splint for the limb.
ambulation training. The patient is having difficulty with standing-up D. instruct the patient in crutch use to protect the joint and
from a seated position due to cocontraction of the quadriceps and assist in early return to walking.
hamstrings during the knee and hip extension phase. The therapist
wishes to use biofeedback beginning with simple knee extension
exercise in the seated position. The plan is to progress to sit-to-stand
training. The initial biofeedback protocol should consist of:
A. high detection sensitivity with recording electrodes placed
closely together.
B. high detection sensitivity with recording electrodes placed
far apart.
C. low detection sensitivity with recording electrodes placed
closely together.
D. low detection sensitivity with recording electrodes placed
far apart.

C167 C2 2
Examination of a patient recovering from stroke reveals a loss of pain
and temperature sensation on the left side of the face along with loss
of pain and temperature sensation on the right side of the body. All
other sensations are normal. The therapist suspect a lesion in the:
A. left cerebral cortex or internal capsule.
B. left posterolateral medulla or pons.
C. midbrain.
D. right cerebral cortex or internal capsule.

C168 C2 2
A patient is taking the drug Baclofen to control spasticity following
spinal cord injury. This medication can be expected to decrease
muscle tone and pain. Adverse reactions of concern to the physical
therapist can include:
A. drowsiness and muscle weakness.
B. headache with visual auras.
C. hypertension and palpitations.
D. urinary retention and discomfort.

C169 C2 2
Symptoms of dysdiadochokinesia, dysmetria, and action tremor can
be expected with a lesion located in the:
A. neocerebellum.

478026779.doc
24 Practice Questions

C172 C8 2 ultrasound unit only has a 5 cm2 soundhead, the therapist should
A 72 year-old male patient recovering from a fractured hip repaired treat:
with ORIF has recently been discharged home. During a home visit, A. each side allotting five minutes for each section.
his wife tells the therapist he woke up yesterday morning and told her B. each side allotting two and a half minutes for each section.
he couldn’t remember much. Upon examination, the therapist finds C. the entire area in five minutes.
some mild motor loss in his right hand and anomia. The therapist D. the entire area in ten minutes.
affirms the presence of short-term memory loss. The BEST course of
action is to:
A. advise the family to document and record any new
problems they notice over the next week, then report back
to the therapist.
B. ignore the findings as they are expected following surgical
anesthesia.
C. refer him to his physician as the therapist suspects a small
stroke.
D. refer him to his physician as the therapist suspects
Alzheimer’s dementia.

C173 C6 3
A 16 year-old with a 4-year history of Type I diabetes is insulin
dependent. He wants to participate in cross country running. The
physical therapist working with the school team advises the athlete to
measure his plasma glucose concentrations before, and after running.
In addition, the student should:
A. avoid carbohydrate-rich snacks within 12 hours of the race.
B. consume a carbohydrate after the race to avoid
hyperglycemia.
C. consume a carbohydrate before or during the race to avoid
hypoglycemia.
D. increase insulin dosage immediately before running.

C174 C6 3
A 61 year-old patient with a transtibial amputation has a short residual
limb. The BEST choice for prosthetic replacement is a:
A. supracondylar suspension.
B. supracondylar/suprapatellar suspension.
C. Syme’s suspension.
D. thigh corset.

C175 C6 1
A physical therapist assistant is ambulating a patient using a three-
point crutch gait. The patient is unsteady and fearful of falling. The
patient does not appear to understand the correct gait sequence. The
supervising therapist’s BEST strategy is to:
A. instruct the PTA to have the patient sit down and utilize
mental practice of the task.
B. instruct the PTA to use a distributed practice schedule to
ensure patient success.
C. intervene and teach the correct sequence since the PTA is
apparently unable to deal with this special situation.
D. tell the PTA and patient to stop the ambulation and work on
dynamic balance activities instead.

C176 C1 1
The therapist is reviewing x-rays from a patient with a Grade 2
spondylolisthesis. The BEST radiographic view to observe this bony
anomaly is:
A. anteroposterior.
B. lateral.
C. oblique.
D. posteroanterior.

C177 C5 3
An athlete presents with pain and muscle spasm of the upper back
(C7-T8) extending to the lateral border of the scapula. This
encompasses a 10 x 10 cm area on both sides of the spine. If the

478026779.doc
Practice Questions 25

C178 C5 2 Abnormalities on his ECG include a flat T wave, prolonged QT


A patient with chronic cervical pain is referred to an outpatient physical interval, and depressed ST segment. The therapist suspects:
therapy clinic. Past medical history reveals: appendectomy, 12 years A. hyperkalemia
ago; chronic heart disease; demand-type pacemaker, 8 years ago; B. hypocalcemia
whiplash injury, 2 years ago. Presently the patient complains of pain C. hypokalemia
and muscle spasm in the cervical region. The modality that should not D. hyponatremia
be considered in the case is:
A. hot pack.
B. mechanical traction.
C. transcutaneous electrical stimulation.
D. ultrasound.

C179 C2 2
In posturography, patients who sway more or fall under conditions with
the eyes closed and platform moving (condition 5) or with the visual
surround moving and platform moving (condition 6) are likely to
demonstrate:
A. problems with sensory selection.
B. somatosensory dependency.
C. vestibular deficiency.
D. visual dependency.

C180 C2 2
A patient presents with severe, frequent seizures originating in the
medial temporal lobes. Following bilateral surgical removal of these
areas, he is unable to remember any new information just prior to the
surgery to the present. He cannot recall text he read minutes ago and
cannot remember people he has met repeatedly. These outcomes are
indicative of:
A. a primary deficit from the loss of the amygdala.
B. loss of integration of the temporal lobe with the basal
ganglia and frontal cortex.
C. loss of procedural memory and integration with frontal
cortex.
D. loss of the hippocampus and declarative memory function.

C181 C3 2
A 14 year-old with a body mass index of 30kg/m2 and a history limited
participation in physical activities is referred for exercise training. The
nutritionist has prescribed a diet limiting his caloric intake. The BEST
initial exercise prescription is:
A. 2 daily sessions of 30 minutes at 40-70% VO2max
B. 2 weekly, sessions of 60 minutes at 50% VO2max
C. 3 weekly sessions of 50 minutes at 70-85% VO2max
D. 3 weekly, sessions of 30 minutes at 60-70% VO2max

C182 C3 2
While on a home visit, a 9 month-old infant becomes unresponsive.
The therapist tell the mother to call for emergency medical services.
The therapist’s IMMEDIATE next step is to tilt the infant’s head back
and give 2 full breaths, while covering both the mouth and nose. Chest
compressions should then be given. The correct procedure for chest
compressions for the infant is to:
A. compress with one hand to a depth of one third to one half
the depth of the chest at a rate of 100/min.
B. compress lightly with 2 fingers to a depth of 2 inches at a
rate of 60/min.
C. compress with both hands to a depth of 1 inch at a rate of
60/min.
D. compress lightly with 2 fingers to a depth of 1/2 to 1 inch at
a rate of 100/min.

C183 C4 2
A patient in an exercise class develops muscle weakness and fatigue.
Examination reveals leg cramps and hyporeflexia. He also
experiences frequent episodes of postural hypotension and dizziness.

478026779.doc
26 Practice Questions

C184 C8 2 A. sinus rhythm with downsloping ST segment depression.


A 64 year-old patient with advanced coronary artery disease and B. sinus rhythm with SA blocks.
diabetes is receiving functional mobility training in a P.T. clinic. While C. sinus rhythm with upsloping ST segment depression.
walking after lunch, the patient experiences difficulty breathing, starts D. tachycardia with abnormal P waves.
hyperventilating, and suffers an episode of syncope. The MOST
LIKELY cause of this problem is:
A. coronary artery disease.
B. hyperglycemia.
C. postprandial hypertension.
D. seizures.

C185 C6 3
The MOST appropriate position to guard a patient who is descending
stairs for the first time using crutches and non-weightbearing on the
right is to:
A. stand behind and slightly to the right side.
B. stand behind and slightly to the right side.
C. stand in front and slightly to the left side.
D. stand in front and slightly to the right side.

C186 C6 3
A patient with paraplegia at the T10 level is being discharged home.
The rehab team is assisting the patient’s wife in modifying the home to
be barrier-free. A new first floor bedroom and bathroom wing are
being built. The builder asks the therapist about the entrance. The
therapist recommends:
A. a ramp with a grade of 12% and a level landing at top of at
least 50 by 50 inches.
B. a ramp with a slope of 1:12 with a level landing at the top of
at least 60 by 60 inches.
C. door width of 36 inches with an outside door that opens
out.
D. door widths of at least 40 inches with a handrail 34 inches
high.

C187 C2 3
A patient has a 10 year history of Parkinson’s disease and has been
on levodopa for the past 6 years. He has fallen 3 times in the past
month resulting in a Colles’ fracture. The therapist decides to try
postural biofeedback training using a platform balance training device.
The training sequence should focus on:
A. decreasing the limits of stability and improving anterior
weight displacement.
B. decreasing the limits of stability and improving posterior
weight displacement.
C. increasing the limits of stability and improving anterior
weight displacement.
D. increasing the limits of stability and improving center of
pressure alignment.

C188 C3 2
A patient is recovering from myocardial infarction and is referred for
supervised exercise training. While working out on a treadmill, the
patient begins to develop mild shortness of breath. Upon inspection of
the ECG readout the therapist determines:

478026779.doc
Practice Questions 27

C189 C8 3 C194 C2 2
A 16 year-old boy with Duchenne’s muscular dystrophy has been A patient recovering from a CVA presents with predominant
confined to using a power wheelchair for the past three years and is involvement of the contralateral lower extremity and lesser
beginning to develop a 10 degree Cobb angle scoliosis. The BEST involvement of the contralateral upper extremity. The patient also
strategy to help slow this spinal curvature at this time would be to: demonstrates mild apraxia. These clinical manifestations are
A. alternate the side of the wheelchair power control. characteristic of:
B. emphasize spinal extension exercises. A. anterior cerebral artery syndrome.
C. emphasize spinal rotation exercises. B. basilar artery syndrome.
D. order a new wheelchair with a reclining seat back. C. middle cerebral artery syndrome.
D. posterior cerebral artery syndrome.
C190 C1 2
A patient presents with a complaint of severe neck and shoulder pain
of two days duration. The patient reports falling asleep on the couch
watching TV, and has been stiff and sore since. There is tenderness
of the cervical muscles on the right, with increased pain upon
palpation. PROM is most limited in flexion, then sidebending left, and
then rotation left, and active extension. Sidebending right and rotation
right is also painful. Based on these examination findings, the patient’s
diagnosis is:
A. cervical radiculopathy.
B. cervical strain.
C. facet syndrome.
D. herniated disc.

C191 C6 2
A postal worker (mail sorter) complains of numbness and tingling in
his right hand in the median nerve distribution. When the therapist
evaluates his work tasks, the therapist notes that he is required to key
in the zip codes of about 58 letters per minute. An appropriate
administrative control to decrease his exposure would be to:
A. provide him with a height adjustable chair to position his
wrists and hands in a neutral alignment.
B. provide the worker with a resting splint to support his wrist.
C. require the worker to attend a cumulative trauma disorder
educational class.
D. use job rotation during the workday.

C192 C5 3
A patient with a traumatic brain injury presents with hemiparesis of the
left upper extremity. The examination reveals slight cutaneous and
proprioceptive impairment, fair (3/5) strength of the shoulder muscles
and triceps and slight spasticity of the biceps. Voluntary control of the
patient’s left arm has not progressed since admission. The therapist
decides to use functional electrical stimulation and place the active
electrode on the triceps to facilitate active extension of the elbow. The
timing sequence BEST to apply is:
A. 2-second ramp up, 10-second stimulation, no ramp down.
B. 2-second ramp up, 5-second stimulation, 2-second ramp
down.
C. 5-second ramp up, 5-second stimulation, 5-second ramp
down.
D. no ramp up, 10-second stimulation, 2-second ramp down.

C193 C2 2
Following a traumatic brain injury, a patient presents with significant
difficulties in learning new skills. He is wheelchair dependent and will
need to learn how to transfer (a skill he has never done before). The
BEST strategy to enhance his motor learning is to:
A. provide bandwidth feedback using a random practice
schedule.
B. provide consistent feedback using a blocked practice
schedule.
C. provide summed feedback after every few trials using a
serial practice schedule.
D. use only guided movement to ensure correct performance.

478026779.doc
28 Practice Questions

C195 C3 1 C198 C6 3
A patient presents with significant intermittent claudication with onset The rehabilitation team is completing a home visit to recommend
after two minutes of walking. On further examination, the therapist environmental modifications for a 72 year-old patient who is scheduled
would expect to find: to be discharged next week. He is wheelchair dependent. The
A. bright red appearance of the extremity in both gravity bathroom has not been adapted. Which of the following
dependent and independent positions. recommendations is NOT appropriate?
B. elevation-induced pallor and dependent redness with the A. adding a tub seat.
extremity in the gravity dependent position. B. adding horizontal grab bars positioned at 45 inches.
C. grayish-white appearance of the extremity in both gravity C. raising the toilet seat to 18 inches.
dependent and independent positions. D. taking the door off to widen the entrance to 32 inches.
D. little or no changes in color with changes in extremity
position. C199 C6 2
During gait, the lateral pelvic tilt on the side of the swing leg, observed
C196 C4 3 during frontal plane analysis, serves to:
Following her cesarean section, a patient tells the therapist that she is A. control forward and backward rotations of the pelvis.
anxious to return to her pre-pregnancy level of physical activity B. reduce knee flexion at mid stance.
(working out at the gym 3 days a week and running 5 miles every C. reduce peak rise of the pelvis.
other day). The therapist’s BEST advice is to tell her to resume D. reduce physiological valgum at the knee.
activities with:
A. a walking program progressing to running after 5 weeks. C200 C1 2
B. abdominal crunches with return to running after 1 month. A patient experiences central thoracic pain while sitting which
C. pelvic floor and gentle abdominal exercises for the first 4-6 increases during the day. There are no complaints in the night or
weeks. morning. Lifting and carrying heavy objects increase the pain. Based
D. pelvic floor exercises and refrain from all other exercise on the above information the MOST LIKELY diagnosis would be:
and running for at least 6-8 weeks. A. herniated nucleus pulposus of the spine.
B. spondylodiscitis.
C197 C8 2 C. thoracic postural syndrome.
A 77 year-old patient lives alone and is referred for home physical D. traumatic compression fracture.
therapy services to improve functional mobility. He refuses to get out
of his chair. Upon examination, he appears irritable, with poor
concentration and memory. He tells the therapist he has not been
sleeping well and has no energy or desire to do anything. He appears
anorexic. The MOST LIKELY explanation for his symptoms is:
A. alcohol abuse.
B. depression.
C. hypothyroidism.
D. pseudodementia.

C1 C3 I
Crackles and cough.
Patients who present with a myocardial infarction and congestive
heart failure have changes to their pulmonary exam, the most
common being crackles and dry cough. Inspiratory wheezing occurs
with extreme airway narrowing, which is not a hallmark of congestive
heart failure. The cough associated with congestive heart failure is
most likely nonproductive. Clubbing of the digits is a sign of chronic
hypoxia. In this case scenario, this patient presents with his first

478026779.doc
Practice Questions 29

myocardial infarction. It is not possible for any chronic changes to A mission statement is a fundamental document that gives direction to
have taken place at this point. all members of an organization as they carry out their current duties
and plan for the future. It describes the organization's purpose; what
C2 C8 I the organization produces, sells, who the customers are, and to some
3 at 10 minutes degree, how people are treated by members of the organization. The
The Apgar Score is based on heart rate, respiration, muscle tone, values statement is the organization's proclamation regarding what
reflex irritability (grimace) and color (appearance). Apgar scores are behaviors are right and wrong, or acceptable or not. The vision
routinely assigned at 1 and 5 minutes and occasionally at 10 min. post statement is inspirational to look forward to what can be achieved and
birth. Scores of 8-10 at 1 minute post birth are normal; scores to enlist support and efforts to make the envisioned possibilities
between 0-3 at 1 and 5 minutes are extremely low and indicative of realities. Strategic plans identify strategies used to meet the overall
the need for resuscitation. Neurological complications are likely with organizational mission.
extremely low Apgar scores, particularly at 10 minutes.

C3 C7 III

Faulty circuitry.
The physical therapist, in this case, correctly delegated the ultrasound
treatment to the PTA. Every individual (PT, PTA) is liable for their own
negligence; however supervisors may assume liability of workers if
they provide faulty supervision or inappropriate delegation of
responsibilities (not evident in this case). Faulty circuitry is the correct
answer. Physical therapists are not liable for defective equipment
unless they contributed to its malfunction, e.g. dropping the ultrasound
machine and continuing to use it in treatment without having it
checked. The institution assumes liability if the patient was harmed as
a result of an environmental problem. The standard of practice is such
that a ground fault interrupter and adequate ultrasound gel are used
during administration of ultrasound.

C4 C6 III
1:1:5
A rest interval equal to one and a half times the work interval allows
exercise to begin before recovery is complete, thus effectively
stressing the aerobic system. The range of effective work-recovery
ratio for aerobic training is 1:1 to 1:5.

C5 C1 III
Pendulum exercises
This individual will typically be immobilized with a sling for a period of
6 weeks. After one week the sling should be removed to have the
patient perform the pendulum exercise to prevent shoulder stiffness.
Resistive exercises are not indicated during this early period. Heat
modalities may be effective in reducing pain but do not improve
mobility.

C6 C1 III
Multifidi working from full flexion back to neutral.
Performing strengthening exercises to the multifidi from flexion to
neutral will not stress the pars defect. Abdominal strengthening will
not provide the segmental stability needed with this condition. Lumbar
extension beyond neutral and rotation will tend to aggravate the
condition in the early stages of rehabilitation.

C7 C1 I
Excessive subtalar pronation.
Possible compensatory motions or postures for forefoot varus
malalignment include: plantarflexed first ray, hallux valgus, excessive
midtarsal or subtalar pronation or prolonged pronation; excessive
tibial, tibial and femoral, tibial, femoral and pelvic internal rotation,
and/or all the contralateral lumbar spine rotation.

C8 C7 II
Mission statement
There are four fundamental documents that would provide guidance
and direction to all members of the organization. They are the values
statement, vision statement, mission statement, and the strategic plan.

478026779.doc
Answers 33

C9 C5 III Cystic fibrosis is an inherited disorder affecting the exocrine glands of


Hamstrings immediately before the quadriceps to produce the hepatic, digestive, respiratory systems. The patient with CF is
cocontraction. prone to chronic bacterial airway infections and progressive loss of
Stimulating the hamstrings just prior to stimulating the quadriceps will pulmonary function from progressive obstructive lung disease. Early
stabilize the tibia and prevent anterior tibial translation during knee clinical manifestations include an inability to gain weight despite
extension and during cocontraction of both muscles. This would excessive appetite and adequate caloric intake. Excessive weight
prevent the moving tibia from placing tension on the injured anterior gain is rarely a finding in a patient with cystic fibrosis. All the other
cruciate ligament. choices are typically present along with persistent coughing,
wheezing, and reduced exercise tolerance.
C10 C2 I
The right DTR is normal while the left is exaggerated. C18 C8 I
Deep tendon reflexes are graded on a 1-4 scale. A 2+ score for DTRs Turn the head and bring the hand to mouth on the same side.
is normal. Abnormal scores include: 0 (absent); 1+ decreased; 3+ ATNR causes extension of upper extremity on the side the head is
exaggerated or hyperactive; and 4+ clonic. Scores of 3+ and 4+ are turned toward. Bringing the hand to the mouth would not be possible
indicative of spastic hypertonia. with an obligatory reflex.

C11 C3 I
Checking resting BP and HR in sitting, then repeating measurements
after standing for 1 minute.
Orthostatic hypotension is a fall in BP with elevation of position, i.e.,
from supine to sitting or sitting to standing. A small increase or no
increase in heart rate upon standing may suggest baroreflex
impairment. An exaggerated increase in HR upon standing may
indicate volume depletion.

C12 C4 III
Document the finding and continue with treatment.
A greenish tinge to the dressing is expected with the use of Panafil.
Panafil is a keratolytic enzyme used for selective debridement. A
greenish or yellowish exudate can be expected. If the exudate was
green and had a foul smell, Pseudomonas aeruginosa should be
suspected and acetic acid would be the topical agent of choice. A
total contact cast can only be used once the wound is free of necrotic
tissue.

C13 C8 I
Turning around and sitting down in a chair
Most falls occur during normal daily activity. Getting up or down from a
bed or chair, turning, bending, walking, climbing/descending stairs all
are high risk activities. Only a small percent of individuals fall during
clearly hazardous activities (e.g. climbing the stepstool). Proper use of
an assistive device reduces the risk of falls.

C14 C6 I
Anterior and lateral walls are 2 ½ inches higher than the posterior and
medial walls.
The anterior and lateral walls are built 2 ½ to 3 inches higher than the
posterior and medial walls to ensure proper positioning on the ischial
seat.

C15 C7 III
Their degree of anxiety and attention.
A needs assessment should include a determination of the level of
anxiety and ability to attend to the instructions given. While the other
factors may also be considered, they do not represent immediate
priorities for hospital-based instruction.

C16 C7 III
95%
In a normal distribution (bell-shaped curve), 95% of scores can be
expected to fall within +2 or -2 SD of the mean; 99% of scores fall
within +3 or-3 SD of the mean.

C17 C3 I
Excessive appetite and weight gain.

478026779.doc
34 Answers

C19 C4 III An appropriate initial exercise prescription for an asymptomatic


To discuss concerns with the physician. individual is 60-90% of HRmax, which is equivalent to 50-85% of V02max
The therapist should respond empathetically and use the moment to or 50-85% of heart rate reserve (Karvonen formula).
talk to the patient about his illness. However the prognosis should not
be discussed as that is the domain of the physician. C27 C3 II
Have him sit down and continue to monitor his vital signs carefully.
C20 C1 I If the chest pain (angina) is exercise induced, this is an indication to
Standing while performing wall push-ups. terminate the exercise session (myocardial demand is exceeding
The long thoracic nerve supplies the serratus anterior muscle. With a myocardial oxygen supply). Recovery is expected after a period of
muscle grade of 3+/5, the patient can then begin functional rest. If the patient is still anxious after the rest, it is reasonable to
strengthening using standing wall push ups using resistance provided return the patient to his room and inform the nurse. This should be
by the patient’s own body. The other exercises would not be optimal done by the therapist personally in order to carefully monitor his
for strengthening a Fair plus serratus anterior. status. The PTA should not be expected to evaluate his chest pain or
reach a determination about its significance. This is not an emergency
C21 C1 II situation.
Left posterior rotated innominate.
A posterior rotated innominate is a unilateral iliosacral dysfunction.
The question outlines positive physical .findings, both static and
dynamic, found with this dysfunction. One of these positive findings
alone does not confirm the diagnosis of left rotated posterior
innominate.

C22 C7 III
The capitation payment method.
Capitation payment methodology allows the managed care insurer to
shift some of the financial risk of care delivery to the selected health
care provider. The provider agrees to accept a set fee in return for the
delivery of services to a specified group of managed care enrollees.
Under the cost-based and fee-for-service payment method the level of
risk is low. The payment risk factors are: charges set below costs,
costs exceed caps, and non-payment of co-payments and
deductibles. Under the per diem payment method the level of risk is
medium. The payment risk factors are: per diem rate set below costs,
services included under the per diem rate are not specifically defined,
high potential for variation in case required, length of service too short,
providers inability to control daily service utilization. Under the
capitation payment method the risk is high. The payment risk factors
include: capitation rate is set below costs, service utilization
projections are lower than actual utilization, providers lack complete
control over the delivery of services, and providers inability to control
service utilization.

C23 C5 III
Brief intense TENS.
Brief intense TENS is used to provide rapid onset, but short-term relief
during painful procedures. The pulse rate and pulse duration are
similar to conventional TENS, however the current intensity is
increased to the patient’s tolerance.

C24 C2 III
Pelvis.
Modification of the pelvic position in a neutral posture promotes good
lumbar and trunk alignment. Many postural problems are correctable
by aligning the pelvis first and achieving a stable base.

C25 C2 II
Chop, reverse chop with right arm leading.
Both chop and reverse chop move the affected arm out-of-synergy.
Thrust is an out-of-synergy pattern while reverse thrust is in-synergy.
Lift is an out-of-synergy pattern while reverse lift is in-synergy. The
same is true for bilateral symmetrical D2F (out-of-synergy) and D2E
(in-synergy).

C26 C3 II
60-90% HRmax

478026779.doc
Answers 35

C28 C4 III pain and complains of instability. The PT intervention does not seem
AROM exercises and walking in a therapeutic pool. to be of benefit. Discontinuing treatment without informing the
Scleroderma (progressive systemic sclerosis) is a chronic, diffuse physician or patient would be inappropriate.
disease of connective tissues causing fibrosis of skin, joints, blood
vessels and internal organs. Patients typically demonstrate symmetric C35 C1 II
skin thickening and visceral involvement of the GI tract, lungs, heart Strengthening the middle and lower trapezius and stretching of
and kidney along with hypersensitivity to touch. The best choice for pectoral muscles.
initial intervention is to exercise in the pool. The warmth and Abnormal posture that produces excessive internal rotation of the
buoyancy of the water will enhance the patient’s movements and shoulders may result in chronic shoulder impingement syndrome due
decrease pain. The other choices are too aggressive at this time and to a loss of scapular stability with overhead motion. Shoulder pain is
risk increasing the patient’s pain, thereby limiting any benefits in likely to continue until a balance between anterior and posterior trunk
flexibility and endurance. musculature is achieved. The anterior chest muscles (pectorals) are
shortened and need stretching and posterior trunk muscles (middle
C29 C8 II and lower trapezius) are stretched and need strengthening.
Language comprehension problems.
Early Stage I Alzheimer’s disease is characterized by memory loss,
absentmindedness, difficulty concentrating and occasional word-
finding problems. Profound communication deficits (inability to speak
or eat), global deterioration of mental functions (delusions,
hallucinations, fragmented memory), agitation, and pacing
(sundowning) are all characteristic of late stages of this disease.

C30 C6 II
Marching while sitting on a therapy ball.
Hip hiking is a compensatory response for weak hip and knee flexors
or extensor spasticity. Active exercises for the hip and knee flexors
(marching) is the most appropriate intervention. Downward manual
pressure on the pelvis strengthens hip hikers. The other choices
focus on strengthening hip and knee extensors.

C31 C7 III
Have him practice locking the brakes first with his left hand, and then
his right.
Transfer training is appropriate to use with this patient. Practice is
performed with less affected extremity first and then progressed to
practice with the more affected extremity. Guided movement (manual
or verbal) represents a less active approach than the transfer of
training approach. The more passive the performance, the slower the
learning. Bilateral tasks are more difficult than performing the task with
one limb.

C32 C7 III
PNF contract-relax technique
The independent variable is the activity or factor believed to bring
about a change in the dependent variable. The type of exercise being
investigated is PNF contract-relax technique. The dependent variable
is the difference in behavior that results as a result of the intervention
(independent variable); in this case, ROM.

C33 C3 I
Decreased AP to lateral chest ratio.
The barreled chest commonly seen in emphysema is measured as an
increased A-P to lateral ratio. Cor pulmonale is hypertrophy or failure
of the right ventricle and results from disorders of the lungs,
pulmonary vessels, or chest wall. Clubbing (bulbous, shiny fingertips
and toes) and cyanosis (bluish or grayish skin) are both present with
chronic lung disease.

C34 C1 III
Discontinue the treatment; and discuss the situation with the patient
and his referring physician.
According to the Guide for Professional Conduct, "when physical
therapists judge that an individual will not benefit from their services,
they shall so inform the individual receiving the services. Physical
therapists shall avoid over-utilization of their services". Consultation
with his referring physician is indicated. After 4 weeks, this patient is in

478026779.doc
36 Answers

50 out of a possible 56 points also indicates low fall risk. A Timed


C36 C4 II GUG score under 20 seconds for the 3 meter walk and turn test
Wound irrigation with pressures below 15 psi. indicates low fall risk (score of over 30 seconds indicate increased
If the ulcer is clean, whirlpool can damage incipient granulation tissue risk).
and should be discontinued. Wet-to-dry dressings help remove
necrotic tissue. Calcium alginate is used in the presence of heavy C43 C6 III
exudates, which is not the case here. Hydrogel would be best as it is A Trendelenburg gait is characterized by a lateral trunk lean toward
non-adherent, keeps wound moist and protects granulation buds; the side of the hip abductor weakness. It is a compensatory strategy
however, whirlpool is inappropriate. Low pressure wound irrigation to keep the pelvis from dropping on the contralateral side during
helps to decrease colonization and prevent infection. stance. The best intervention is bridging which activate both hip
extensors and abductors. The use of Theraband increases the loading
C37 C1 II of hip abductors. Lateral leg slides (gravity eliminated position for hip
Extension abductors) would not sufficiently strengthen the hip abductors for
With spondylolisthesis there is typically an anterior slippage of one weightbearing function. The other choices do not encourage static
vertebra on the vertebra below. Because of the anterior shearing holding of hip abductors.
forces acting at the vertebra caused by the wedge shape of the
vertebra and gravity, spinal extension positions should be avoided.

C38 C2 II
Carpal tunnel syndrome
The pattern of motor and sensory loss corresponds to the median
nerve distribution in the hand. The most likely cause is carpal tunnel
syndrome. Pronator teres syndrome (also a median nerve problem)
produces similar deficits along with involvement of the flexors of the
wrist and fingers. Cervical root compression would also produce
proximal deficits in strength and sensation.

C39 C2 II
Guillain-Barré syndrome
These signs and symptoms are characteristic of Guillain-Barre
syndrome, a peripheral neuropathy in which there is inflammation and
demyelination of peripheral motor and sensory nerve fibers. Early in
its progression either upper or lower motor signs may predominate. In
almost all cases, patients with ALS show features of both UMN and
LMN dysfunction. Post-polio syndrome is a lower motor neuron (LMN)
syndrome that does not present with sensory paresthesias and is
typically asymmetrical. MS will present with upper motor neuron signs:
spasticity and hyperreflexia.

C40 C3 III
Slowing of pulse and increased venous pressure are possible.
Valsalva maneuver results from forcible exhalation with the glottis,
nose, and mouth closed. It increases intrathoracic pressures and
causes slowing of the pulse, decreased return of blood to the heart,
and increased venous pressure. A cholinergic or vagal response is the
result of parasympathetic nervous system (PNS) stimulation.

C41 C4 II
Arterial insufficiency
Arterial insufficiency can be determined by skin color changes during
position changes of the foot, termed rubor of dependency test.
Chronic venous insufficiency can be determined by the history,
presence of aching calf pain with prolonged standing, a percussion
test in standing, or Trendelenburg's test (retrograde filling test). With
chronic insufficiency, skin will be dark and cyanotic. Acute deep vein
thrombophlebitis can be evident with aching calf pain, edema, and
muscle tenderness. Lymphedema is evident with visual inspection
(i.e. swelling, decreased ROM) and volumetric measurements.

C42 C8 II
Functional Reach of 7 inches.
All of these instruments can be used to examine functional balance
and fall risk. A Functional Reach score of less than 10 is indicative of
increase fall risk. A POMA score of 27 out of a possible 28 is an
excellent score (scores below 19 indicate a high risk for falls while
scores between 19-24 indicate moderate risk for falls). A Berg score of

478026779.doc
Answers 37

C44 C3 II
Not use supplemental O2 C52 C3 II
A 70 year-old male would likely have a resting oxygen saturation of A 2o AV heart block.
95% from the changes associated with aging alone. There is no need Criteria for reducing exercise intensity according to the American
to supplement oxygen in this case. The guideline for supplemental College of Sports Medicine include: 1) onset of angina and other
oxygen is an SaO2 < 88%. Therefore, the use of oxygen in this symptoms of exertional intolerance, 2) SBP equal to or greater than
scenario is not justified. Supplemental O2 is by prescription only 240 mm Hg, DBP equal to or greater than 110 mm Hg, 3) greater than
unless it is an emergency. 1 mm ST segment depression, horizontal or downsloping, 4)
increased frequency of ventricular arrhythmias, 5) 2° or 3° AV block,
C45 C8 II or other significant ECG disturbances.
Lightweight wheelchair.
Ambulation at this level lesion requires too much energy and time to C53 C4 III
be functional. The parapodium permits standing but does not allow for Begin with an interval walking program, exercising only to the point of
sufficient mobility for the entire school day. The lightweight wheelchair pain.
is the best choice for this child. This patient is exhibiting classic signs of chronic arterial insufficiency.
Rehabilitation guidelines for arterial disease include using an interval
C46 C1 III walking program of moderate intensity and duration, 2-3 times/day.
Because the left thoracic facet joint capsule is restricting movement, The patient should be instructed to exercise to the point of pain, not
motion that would stretch the capsule would facilitate improved right beyond. Exhaustive exercise and persistent pain are contraindicated.
rotation. With right rotation the left superior facets move upward Calcium channel blockers may be used in vasospastic disease;
(opening the joint and stretching the capsule) and right facets move exercise is not contraindicated.
downward (closing the joint and putting the capsule on relative slack). C54 C8 III
Additionally, trunk flexion also moves the superior facets upward Clean and debride the wound, and apply a hydrogel dressing.
(opening the joint and stretching the capsule). Therefore, combined A stage II ulcer (deep ulcer) involves a partial thickness skin loss with
right rotation and flexion will most effectively stretch the left facet joint involvement of epidermis, dermis or both; it is reversible. Intervention
capsule. should be directed toward improving perfusion and relieving localized
pressure. The wound should be cleaned with an antimicrobial agent,
C47 C1 I debrided of necrotic tissue, and covered with a sterile dressing.
Soleus and gastrocnemius. Hydrogel dressings maintain moisture in the wound bed, soften
The soleus and gastrocnemius muscles oppose the dorsiflexion necrotic tissue, and support autolytic debridement. Pressure relief is
moment that exists at the ankle due to the line of gravity which falls also an important consideration. Techniques of protective foot care
slightly anterior to the lateral malleolus. should be taught. A dry, sterile dressing is contraindicated as is the
application of skin lubricants.
C48 C7 III
What occurred, when and where it occurred and witness statements. C55 C6 III
An incident report should avoid interpretive information such as cause A 17.5 inch seat height.
of the occurrence or corrective actions that were taken. The typical A hemi or low seat wheelchair has a seat height of 17.5 inches (20
information included on an incident report is the name of those inches is the standard seat height). The lower seat height permits the
involved, inclusive of witnesses, what occurred, when it occurred and patient to propel and steer the wheelchair using the sound upper and
where it occurred. There is no presumption that someone was injured. lower extremities. Elevating legrests may be considered if the patient
has problems with edema but should not be considered standard.
C49 C5 III Detachable arms are a useful option but not the most appropriate in
US at 1 MHz this example.
Pulsed ultrasound produces non-thermal effects (acoustic streaming,
cavitation, microstreaming) which during the early stages of C56 C3 III
inflammation process promotes tissue repair. The therapist would Give 2 rescue breaths followed by 30 chest compressions, repeating
select 1 MHz for a deep therapeutic effect. the cycle.
New guidelines form the American Heart Association (November
C50 C2 II 2005) concerning Basic Life Support specifies calling 911 for
This patient demonstrates involvement of the long tracts (sensory and unresponsive adults before beginning CPR. The compression rate for
motor) indicative of involvement of the contralateral cerebral cortex. adult CPR is about 100 per minute with a recommended compression-
The involvement of the face indicates a lesion above the level of the to-ventilation ratio of 30:2. The old ratio was 15:2. Lay rescuers will
midbrain. A lesion in the spinal cord would not affect the face. A lesion be taught to no longer check for pulse but to begin chest
in the brainstem would produce facial signs contralateral to the limb compressions immediately. If available, the healthcare rescuer should
signs. retrieve the AED after calling for emergency services and if trained,
use the AED after beginning CPR.
C51 C2 I
Stocking and glove distribution of the lower and upper extremities. C57 C1 II
Symmetrical involvement of sensory fibers, progressing from distal to Descending stairs.
proximal, is the hallmark of polyneuropathy. It is termed stocking and Loss of dorsiflexion will make descending stairs most difficult because
glove distribution and is the result of the dying back of the longest the ankle must have dorsiflexion during the single limb support phase
fibers in all the nerves from distal to proximal. Sensory symptoms during descent. Full range in dorsiflexion is not needed for the other
include decreased sensation and pain, paresthesias and dysesthesias choices.
(abnormal sensations such as numbness, tingling, or prickling).
Allodynia refers to the perception of an ordinarily painless stimulus as C58 C6 III
painful and is not characteristic of polyneuropathy.

478026779.doc
38 Answers

Higher than the torque actually generated by the contracting


hamstrings.
Gravity produced torque adds to the force generated by the
hamstrings when it contracts, giving a higher torque output than is
actually produced by the muscle. Testing values may be misleading;
software is available to correct for the effects of gravity.

C59 C1 III
Central P/A pressure at a 60 degree angle on the spinous process of
T6 while stabilizing T5.
In a closing restriction, the inferior facets of the superior vertebra will
not inferiorly glide on the superior facets of the inferior vertebra.
Therefore T5 inferior facets will not caudally glide on the superior
facets of T6. Stabilizing T5 and application of pressure to T6 localizes
the cephalad movement of the superior facets T6 on T5 bilaterally.
The angle of the thoracic facets is 60 degrees, therefore the
application of force should be at the same plane.

C60 C2 II
Benign paroxysmal positional vertigo.
Benign paroxysmal positional vertigo (BPPV) is characterized by
acute onset of vertigo and is positional, related to the provoking
stimulus of head movement. Vestibular neuritis is an inflammation of
the vestibular nerve caused by a virus and typically produces
symptoms of dysequilibrium, nystagmus, nausea and severe vertigo.
Meniere's disease is characterized by a sensation of fullness in the
ears associated with abnormal fluid build-up. Additional symptoms
include tinnitus, vertigo, nausea and hearing loss. Acoustic neuroma
(vestibular schwannoma) produces unilateral sensorineural hearing
loss along with vestibular symptoms.

C61 C3 II
Increased heart rate and contractility at rest.
Atropine is an anticholinergic agent (it blocks the action of
acetylcholine at parasympathetic sites in smooth muscle, secretory
glands, and the CNS). It produces an increase in heart rate and
contractility and is used to treat symptomatic sinus bradycardia, and
exercise-induced bronchospasm. Major adverse cardiovascular
reactions can include tachycardia, orthostatic hypotension,
palpitations, ventricular fibrillation, and increased ischemia in patients
with myocardial infarction.

478026779.doc
Answers 39

C62 C4 II Do a comprehensive examination, and if the therapist suspects abuse


Hematuria and ecchymosis. report the finds to the appropriate authorities.
Warfarin sodium (Coumadin) is an anticoagulant, indicated in the According to APTA’s Guidelines for Recognizing and Providing Care
prophylaxis and treatment of venous thrombosis, pulmonary for Victims of Domestic Violence this patient falls into a category of
embolism, and thromboembolic disorders. Potential adverse reactions high risk. Women between the ages of 17 and 28 years and woman
include hematuria and ecchymosis (skin discoloration and who are single, separated, or divorced or who are planning a
hemorrhaging). Serious bleeding is possible with drug toxicity. separation or divorce are at high risk. Battered women usually have
Xeroderma (dry skin), cellulitis (inflammation of tissues) and more than one injury. Most injuries occur in the head, face, neck,
palpitations (awareness of heart rhythm abnormalities) are not seen breasts, and abdomen. According to AMA, battered women represent
as adverse reactions with warfarin. 23% of pregnant women who seek prenatal care. The victim may not
volunteer information about her situation, but more often than not
C63 C8 III when asked they will reveal it. It is important for the physical therapist
Highlighting steps with pastel colors of blues and greens. to identify resources available for victims of domestic violence and to
Cataracts produce a clouding of the lens with gradual loss of vision. know their own state reporting laws. In most state jurisdictions a
Patients experience difficulties in low light, high glare environments, physical therapist may be fined or indicted for failure to report.
and in perceiving cool colors. Color coding with warm colors: reds,
yellows or oranges is appropriate.

C64 C6 II
Standing, partial wall squats.
The quadriceps muscle is responsible for most of the energy
generation needed to transfer up stairs to the next level. Partial wall
squats are the best choice to strengthen these muscles (closed chain
exercise). During forward continuance (corresponding to midstance),
the ankle plantarflexors assist. Hip extensors are also active
concentrically, assisting these actions. The other choices might be
good lead-up activities for gait but would not optimally strengthen the
key muscles involved in ascending stairs.

C65 C4 II
Postpone ambulation and report the findings immediately.
The patient is exhibiting early signs of acute deep vein
thrombophlebitis (DVT). These findings should be reported
immediately. Exercise and ambulation are contraindicated during the
acute phase. If DVT is present, the patient will be given
anticoagulation medication and will remain on bed rest with elevation
of the involved leg until the acute phase subsides.

C66 C8 I
Asking the child and his parents to describe the boy’s most serious
functional limitations.
The child and his parents/caretakers play an important part in
determining impairments, functional limitations, disability and future
interventions. Taking a thorough initial history is important in
determining what the other components of the examination should be.
The incorrect choices, may indeed be appropriate; however, the
interview process may help decide which examination tools are
needed.

C67 C1 I
Anteversion.
The pathology commonly associated with medial femoral torsion and
toeing-in is hip anteversion due to an increase in the angle (greater
than 15 degrees) between the femoral condyles and neck of the
femur.

C68 C1 III
Trunk extension and abdominal stabilization exercises.
Patients with osteoporosis must be educated in the area of prevention.
They should avoid trunk flexion or rotation exercise as it may facilitate
a compression fracture of the spine. They also have a fear of falling
and fractures, therefore it is very important to strengthen from the core
to the floor as well as train in proprioception and balance
enhancement techniques.

C69 C1 II

478026779.doc
40 Answers

C70 C5 III extension exercises are safe and effective secondary to the dynamic
High volt monophasic pulsed current. stability inherent with this type of exercise.
Because high volt pulsed current is a monophasic, unidirectional
current, the unidirectional current would produce a therapeutic effect C76 C4 III
at the active (treatment) electrode. A negative charge (polarity) Facilitate autolytic debridement and absorb exudate.
should be applied for a bactericidal effect of a positive charge given to Moisture-retentive occlusive would dressings such as calcium alginate
promote wound healing. A biphasic current, which alternates the are recommended for use on exudating wounds (grade III ulcer).
polarity, would tend to negate the treatment effects. Russian (burst) They maintain a moist wound environment and do NOT allow rapid
and interferential (beat) are medium frequency biphasic currents. evaporation, absorb exudate, provide autolytic debridement, reduce
Interrupted currents (> 0.5 seconds interruption) are also not used for pain at the wound site, and promote faster healing (reepithelialization).
tissue healing. A disadvantage is that they are very permeable to bacteria, urine, and
so forth. Semirigid dressings such as Unna’s boot provide little
C71 C5 III support.
Prone with no pillow.
Placing the patient in the prone position would better align the spine
so that the pull of the traction would be along the axis of the vertebral
bodies. If this position is intolerable, then a pillow may be placed
under the abdomen. Flexing the spine could exacerbate the disc
herniation. A supine, knee flexed position can be used for spinal
stenosis.

C72 C2 III
Abduction with elbow extension
Hemiplegic synergies are present and should not be reinforced.
Shoulder abduction with elbow flexion is part of the flexion synergy
while adduction with elbow extension is part of the extension synergy.
Adduction with elbow flexion is an out-of-synergy combination but
does not strengthen the deltoid. Shoulder abduction with elbow
extension is the correct choice.

C73 C2 II
The sound side.
Unopposed action of the eye muscles causes the eye to deviate in the
direction of the intact musculature. Thus, patients with hemispheric
lesions may look away from the hemiplegic side. Patients with
brainstem lesions may look toward the hemiplegic side.

C74 C3 II
ST segment depression from baseline of 3 mm horizontal or
downsloping depression.
A positive exercise tolerance test (graded exercise test) indicates
myocardial ischemia with increasing exercise intensities. The optimal
test duration is 8-12 minutes but can be terminated if symptoms of
exertional intolerance are evident. The American College of Sports
Medicine indicates these include: 1) onset of moderate to severe
angina, 2) a drop in SBP with increasing workload, 3) serious
arrhythmias, 4) signs of exertional intolerance (pallor, cyanosis, cold
or clammy skin), 5) unusual or severe shortness of breath, and 6)
CNS signs (ataxia, vertigo, visual or gait problems, confusion).
Relative indications to stop the test include: 1) hypertensive response
(equal to or greater than 260/115 and 2) ECG changes from baseline
(greater than 2 mm horizontal or downsloping; ST segment
depression or greater than 2 mm ST segment elevation). Source:
ACSM’s Guidelines for Exercise Testing and Prescription, 6th ed,
Lippincott Williams & Wilkins.

C75 C1 II
Closed chain terminal knee extension exercises.
The test that was conducted was a Lachman’s test to determine
integrity of the anterior cruciate ligament. A positive test suggests
laxity of the anterior cruciate ligament. Quick cutting/lateral
movements that occur in agility training and heavy joint leading that
occurs with plyometric exercise should be avoided until the muscular
restraints that reduce excessive anterior translation of the affected
tibiofemoral joint are strengthened. Open-chain knee extension may
place excessive load on the ACL. Closed-chain terminal knee

478026779.doc
Answers 41

C77 C4 III Inferior glide at 55 degrees of abduction.


Avoid direct exposure to blood and body fluids. The convex-concave rule for mobilization applies. The most effective
Hepatitis B is a viral infection that is transmitted by close contact with position to mobilize for improved shoulder abduction is in the resting
the infected patient’s body fluids (nasopharyngeal exudate, saliva, position (55 degrees). Because the convex humeral head is moving
sweat, urine, feces, semen, vagina secretions) and blood and blood on the concave glenoid, an anterior glide would be most appropriate to
products. Healthcare workers should be vaccinated against the improve shoulder abduction.
possibility of infection as they are in a high risk category and should
carefully follow standard precautions to protect themselves (refer to
Chapter 5, Table 5-2). Droplet precautions are used when
microorganisms can be transmitted by the patient during coughing,
sneezing or talking. Contact precautions are designed to reduce the
risk of microorganism transmission by direct or indirect contact.

C78 C6 II
Knee extensors.
The quadriceps are maximally active at heel strike (initial contact) to
stabilize the knee and counteract the flexion moment.

C79 C3 II
Have the patient sit down, continue monitoring, and notify the
physician immediately.
This tracing shows premature ventricular contractions (PVCs) that are
multifocal (originating from different irritable ventricular focus). These
multiform PVCs pose a potential danger of deteriorating into
ventricular tachycardia and ventricular fibrillation (cardiac standstill).
Since the heart is demonstrating a high degree of irritability, the best
course of action is to stop the exercise, have the patient sit down,
continue monitoring carefully, and notify medical staff (attending
physician) immediately.

C80 C7 III
Practice stair climbing inside the parallel bars using a 3 inch step.
The most appropriate lead-up activity to promote the skill of stair
climbing is practice using a 3 inch step in the parallel bars. Passive
movements do not promote active learning. Marching in place and
isokinetic training may improve the strength of the hip flexors but do
not promote the same synergistic patterns of muscle activity as the
desired skill.

C81 C3 III
Postural drainage, percussion, and shaking to the right lower lobe.
Viral pneumonia does not cause an intra-alveolar exudate; therefore,
percussion and shaking are not indicated. All of the other
interventions could be beneficial to the patient.

C82 C4 II
Outline realistic short term goals to improve independence while
maintaining for the possibility of further recovery.
This patient is exhibiting denial, the first stage in psychological
acceptance (Kubler-Ross). Denial can be protective, particularly in
the early phases. However, this patient’s denial persists three months
after injury and is limiting her rehab progress and outcomes. It is
important to provide a message of hope tempered with realism.
Encouraging short term goals to improve function is the most
appropriate choice.

C83 C8 III
Reposition the child in a proper sitting position using postural
supports.
Feeding can only be successful if the child is positioned in a stable
sitting posture: head upright, trunk erect with pelvis neutral and hips
flexed to 90 degrees, and feet resting flat. Correct positioning in
sitting will facilitate upper extremity function (grasp and release) as
well as swallowing.

C84 C1 III

478026779.doc
42 Answers

C85 C1 II Abduction and internal rotation of the shoulder places the


Capsule-ligamentous pattern of TMJ on the left. supraspinatus tendon in a good position to apply ultrasound by
The capsule-ligamentous pattern of the TMJ is limitation on opening, exposing the tendon from under the acromion process.
lateral deviation greater to the uninvolved side, and deviation on
opening is to the involved side. Weakness of the lateral pterygoids C89 C2 III
presents as deviation on protrusion to the opposite side of the muscle Reach forward to bear weight with the right arm extended against the
weakness. A capsular pattern of the cervical spine presents as side wall.
flexion and rotation equally limited and extension. Normal parameters Stage 4 is characterized as some movement combination that do not
for TMJ measures are 25-3C8 III follow the paths of either synergy. The influence of spasticity is
Reposition the child in a proper sitting position using postural beginning to decline. Weightbearing on extended arm in sitting is an
supports. early training activity (stage 2-3) to promote shoulder stability while
Feeding can only be successful if the child is positioned in a stable keeping the arm out of its typical spastic pattern. Reaching overhead
sitting posture: head upright, trunk erect with pelvis neutral and hips with the elbow extended is an advanced activity, characteristic of
flexed to 90 degrees, and feet resting flat. Correct positioning in stage 5 recovery. Putting on socks and shoes is also an advanced
sitting will facilitate upper extremity function (grasp and release) as activity for the upper extremity requiring manual dexterity. Reaching
well as swallowing. forward and weightbearing with the shoulder at 900 and elbow
extended is the best choice for stage 4 training.
C84 C1 III
Inferior glide at 55 degrees of abduction.
The convex-concave rule for mobilization applies. The most effective
position to mobilize for improved shoulder abduction is in the resting
position (55 degrees). Because the convex humeral head is moving
on the concave glenoid, an anterior glide would be most appropriate to
improve shoulder abduction.

C85 C1 II
Capsule-ligamentous pattern of TMJ on the left.
The capsule-ligamentous pattern of the TMJ is limitation on opening,
lateral deviation greater to the uninvolved side, and deviation on
opening is to the involved side. Weakness of the lateral pterygoids
presents as deviation on protrusion to the opposite side of the muscle
weakness. A capsular pattern of the cervical spine presents as side
flexion and rotation equally limited and extension. Normal parameters
for TMJ measures are 25-35 mm functional, and 35-50 mm normal,
normal protrusion is 3-6 mm, and normal lateral deviation is 10-15
mm.

C86 C7 III
Deny access to the chart unless written permission by his wife is
granted.
The issue here is patient confidentiality. Spouses do not have access
to medical information unless they have consent of proxy because of
incompetence of the other spouse. In accordance with APTA’s Guide
for Professional Conduct Principle 1. Physical therapists respect the
rights and dignity of all individuals. Specifically 1.1 Attitudes of
Physical Therapists B. Physical therapists are to be guided at all
times by concern for the physical, psychological and socioeconomic
welfare of those individuals entrusted to their care; and 1.2
Confidential Information C. Information derived from the working
relationship of physical therapists shall be held confidential by all
parties. HIPAA regulations also limit access to medical record
information unless consent is granted.

C87 C3 II
Pneumothorax
While all of the pathologies listed would cause panic on the part of the
patient, mucous plugging of an airway would not cause pain. The
deviation of the trachea would not result from angina or pulmonary
emboli, but would happen with a pneumothorax and lung tissue
collapse (which could result from mucous plugging). The deviation of
the trachea toward the right, with the chest pain on the left, is a match
of symptoms for the occurrence of a pneumothorax on the left. The
history of a rib fracture makes pneumothorax all the more likely.

C88 C5 III
Slight abduction and internal rotation.

478026779.doc
Answers 43

C90 C2 II With wound conversion, grafting will be necessary as all epithelial


Broca’s aphasia. cells are destroyed with a full thickness burn.
This patient is demonstrating classic signs of Broca’s aphasia (also
known as nonfluent, expressive, or motor aphasia). It is the result of a C96 C6 III
lesion involving the third frontal convolution of the left hemisphere. Viscoelastic shoe insert with forefoot lateral wedge.
Broca’s aphasia is characterized by slow and hesitant speech with Supination of the foot (pes cavus) is accompanied by supination of the
limited vocabulary and labored articulation. There is relative talocalcaneonavicular (TCN), subtalar and transversal tarsal joints. It
preservation of auditory comprehension. Wernicke’s aphasia is is characterized by an abnormally high arch. The flexible cavus foot
characterized by impaired auditory comprehension and fluent speech. generally responds well to orthotic foot control especially in a young
Global aphasia is a severe aphasia with marked dysfunction across all child. The best choice is a viscoelastic shoe insert with forefoot lateral
language modalities. Dysarthria is an impairment in the motor wedge. The other choices are used to control flexible pes valgus.
production of speech.
C97 C6 III
C91 C2 III Stand pivot transfer to the sound side.
Engage in a calming activity and document the behaviors. During initial healing, it is important to protect the hip from dislocation
Patients in Level IV of recovery are confused and agitated. Behavior is or subluxation of the prosthesis. With a posterolateral incision.
bizarre and nonpurposeful relative to the immediate environment. This excessive hip flexion and adduction past neutral are contraindicated.
patient is unable to cooperate directly with formal examination or This is minimized by transferring to the sound side. Full ROM of the
treatment, lacking both selective attention and memory. The therapist operated hip is also contraindicated.
needs to observe and document the behaviors closely and engage the C98 C6 III
patient in a calming activity such as slow rocking. A quiet, closed A rigid frame.
environment is critical. Sports competition wheelchairs are usually made with rigid
construction and very strong lightweight materials. A folding
C92 C3 I wheelchair does not provide the stability needed for competition
Left ventricular failure. sports. Pneumatic (air-filled) tires provide a smoother ride and
Clinical manifestations of left ventricular failure (congestive heart improved traction as opposed to hard-rubber tires. A low seat back
failure), include: those described in the case example along with an enhances the user's upper body/arm movements. A higher seat back
S3 heart gallop, paroxysmal nocturnal dyspnea, orthopnea, and signs is indicated for patients with decreased trunk control (not a factor in
and symptoms of pulmonary edema (marked dyspnea, pallor, this example).
cyanosis, diaphoresis, tachypnea, anxiety, and agitation). Typical
clinical manifestations of right ventricular failure include: dependent C99 C8 III
edema of the ankles (usually pitting edema), weight gain, fatigue, right Sideward protective extension in sitting.
upper quadrant pain, anorexia, nausea, bloating, right sided S3 or S4, Sideward protective extension in sitting is a functional, protective
cyanosis of nail beds and decreased urine output. Impending MI may reaction which normally occurs at about the same time as sitting
include anginal pain or discomfort in the chest, neck, jaw or arms, begins. The child who is starting to sit should already have prone and
palpitations, tachycardia, or unusual fatigue or dyspnea. Pericarditis supine tilting reactions. It is too early to begin standing tilting
produces substernal pain that may radiate to neck and upper back, reactions.
difficulty swallowing, pain aggravated by movement or coughing and
relieved by leaning forward or sitting upright, and a history of fever, C100 C3 II
chills, weakness, or heart disease. 40 to 85% HR reserve (Karvonen’s formula).
This patient has impaired functional abilities as noted by the low
C93 C3 II number of feet traveled in 12 minutes. He also has a high resting heart
Prolonged endurance training has resulting in a low heart rate. rate, likely the result of his pulmonary medications. To correctly
One of the benefits of endurance training is decreased heart rate and prescribe exercise for this patient, his high resting heart rate needs to
blood pressure with improved functional capacity. This patient be taken into consideration. Karvonen's formula, the heart rate
demonstrates a significant training effect, most likely from months or reserve method, uses his high resting heart rate as part of the formula.
years of consistent running. Low heart rate is not a usual finding with [(HRmax - HRrest) x 40 to 85%] + HRrest. = Target heart rate range. If the
most cardiovascular pathology. therapist were to use the formula 70 to 80% of HRmax, the therapist
would find that part of his target heart rate for exercise was lower than
C94 C4 II his resting heart rate. Using a predicted number for heart rate when an
Atrophy and osteoporosis. actual exercising heart rate max is known is an inaccurate way of
Prolonged use of corticosteroids may result in muscle weakness, prescribing exercise. Finally, since 1) the MET charts are based on a
osteoporosis, fractures and joint pain. Large doses are associated healthy population, 2) the therapist has the actual heart rate data from
with Cushingoid changes (moonface, central obesity, hypertension, the test and, 3) the therapist doesn't have actual metabolic
myopathy, electrolyte and fluid imbalance, and so forth). Common equivalents during the exercise test, there is no reason to use METs in
CNS changes include insomnia and nervousness. the calculation of exercise intensity.

C95 C4 I C101 C8 II
The infected wound can convert the area to a full thickness burn. Independent in wheelchair mobility.
A deep partial thickness burn will heal in about 3-5 weeks if it does not Duchenne's muscular dystrophy is a progressively debilitating disease
become infected. An infection typically results in conversion of the with the majority of teenagers becoming wheelchair bound but
wound to a full thickness burn. Full thickness burns are without independent. A motorized tilt-in space wheelchair is a good choice.
sensation as the nerve endings are destroyed. However, the area is
not pain free as adjacent areas of partial thickness burns have intact C102 C1 I
nerve endings and can experience pain. The risk of hypertrophic and 9-11
keloid scars is high with deep partial thickness or full thickness burns.

478026779.doc
44 Answers

The most effective age to screen girls for scoliosis is just before the present even at rest. The findings cannot explain the incorrect
pubescent growth spurt between 9-11 years when the scoliotic curve choices.
can increase dramatically. Boys should be screened between 11 to 13
years of age due to differences in the age of onset of puberty between C110 C4 II
girls and boys. Discontinue treatment and notify his physician immediately.
This patient is demonstrating signs and symptoms of aortic aneurysm.
C103 C1 III Pain is intermittent or constant and can be lei! in the midabdorninal or
Lower cervical flexion. low back regions. The pulsating mass is highly significant and his level
Flexion increases the space at the intervertebral foramen, allowing the of hypertension dramatically increases his risk of rupture. This is a
C6 nerve root to decompress and reduce or alleviate radicular pain. serious medical condition; you should notify his physician immediately.
You should not rely on the patient to contact his physician. PT
C104 C1 I intervention should cease.
Tibial, femoral, and pelvic internal rotation.
With the patient standing, the calcaneus is fixed to the ground. C111 C4 III
Subtalar joint pronation will occur as the talus plantar flexes, adducts Whirlpool with povidone-iodine.
and inverts. In response to subtalar joint pronation, obligatory internal Wound irrigation or lavage is an effective way to remove debris and
rotation of the tibia, femur, and pelvis occurs. contaminants, and reduce bacterial counts on wound surfaces.
Whirlpool can be used as a means of wound cleansing and
C105 C7 III mechanical debridement. Antibacterial agents are typically used, e.g.,
According to APTA's Guidelines for Physical Therapy Documentation: povidone-iodine (Betadine) or Chloramine-T (Chlorazine) if the wound
Charting errors should be corrected by drawing a single line through is purulent. Limitations of whirlpool include risk of contamination and
the error and initialing and dating the chart. The appropriate use of the dependent position can increase venous congestion and
mechanism for electronic documentation clearly indicates that a edema. Water temperature should be warm (not 20 degrees Celsius)
change was made without deletion of the original record. to stimulate peripheral circulation. Hydrogen peroxide is indicated to
debride wounds with large amounts of necrotic tissue and is not used
C106 C5 III for wound lavage. Dakin’s solution is an appropriate bactericidal
Pulse duration. agent; however, it is not administered while the patient is in the
Decreasing the pulse duration reduces the electrical charge of each whirlpool.
pulse making the current more comfortable by decreasing the total
current applied while maintaining the full therapeutic effect. C112 C8 I
Sundowning behavior
A patient with Stage 2 Alzheimer’s disease, can be expected to exhibit
impaired cognition and abstract thinking, sundowning (defined as
extreme restlessness, agitation, and wandering that typically occurs
C107 C2 II later afternoon), inability to carry out ADL, impaired judgment,
Horner’s syndrome inappropriate social behavior, lack of insight, repetitive behavior and a
Horner's syndrome occurs with lesions of the lateral medulla or voracious appetite. Inability to communicate is characteristic of Stage
cervical sympathetic chain. Symptoms include miosis, ptosis, and 3. Short term memory loss and disorientation to time and date are
anhidrosis over the face. Argyll Robertson pupil is a small, fixed size early signs of the disease (Stage 1).
pupil that does not react to light but does contract with
accommodation. It is caused by neurosyphilitic lesions interrupting C113 C6 III
fibers from parasympathetic nuclei (Edinger-Westphal nuclei) of the Provide posterior directed resistance to the right ASIS during stance.
oculomotor nerve. Homonymous hemianopsia is a field detect Stride length is the linear distance between heel strike of one lower
involving loss of visual information from one hemifield (common in extremity to the next heel strike of the same extremity. Resistance
cortical lesions and stroke). Nystagmus is abnormal oscillating eye applied in a posterior direction to the right ASIS during stance will
movement. facilitate forward pelvic rotation and enhance stride length. Anterior
directed resistance functions to pull the hip forward but does little to
C108 C2 II facilitate active motions. The gluteals function to stabilize the limb.
Vestibulocerebellum
The symptoms are suggestive of cerebellar dysfunction. The C114 C6 III
vestibulocerebellurn (archicerebellum) is concerned with adjustment of Prone-on-elbow pushups.
muscle tone in response to vestibular stimuli. It coordinates muscle Prone-on-elbow pushups strengthen glenohumeral and scapular
actions to maintain equilibrium. The spinocerebellum musculature required for pushups in sitting. While strengthening of the
(paleocerebellum) controls muscle tone and synergy of muscles on triceps (intact in this patient) is also important, achieving proximal
the same side of the body. stability first is critical to promoting control. Supine-on-elbows activities
are used to assist in bed mobility and to prepare the patient to assume
C109 C3 I the long sitting position.
Peripheral arterial disease. C115 C7 III
Intermittent claudication, often the earliest indication of peripheral Level 1.0 Receiving
arterial disease (PAD), is manifested by cramping, pain, or fatigue in In the affective domain, level 1.0 objectives deal with attending to
the muscles during exercise, which is typically relieved by rest. The phenomena and stimuli. This student lacks these foundation behaviors
calf muscle is most commonly affected, but discomfort may also occur in the affective domain and is thus functioning at a very low level. His
in the thigh, hip, or buttock. Cessation of pain immediately upon problem is not one of behaviors at the higher levels of the affective
stopping the exercise is characteristic of intermittent claudication, not domain (all other choices).
other spinal problems. With severe disease however, pain may be

478026779.doc
Answers 45

C116 C7 III
Have the same therapist reassess the patients after 6 months.
Reliability is the degree to which a test consistently measures what it
is intended to measure. Intrarater reliability is established by having
the same rater measure on multiple measurement trials. Intrarater
reliability can be established if all raters are trained to administer the
measurement instruments and demonstrate consistency of rating
(there is no evidence of this).

C117 C3 II
Heart rate.
An exercise tolerance test should be performed prior to commencing
an exercise program for all high-risk individuals. The best
measurement of exercise intensity in a newly exercising individual is
heart rate. Rating of perceived exertion (RPE) will become a valuable
measurement tool once the patient becomes adept at using it, but it
would not be reliable for the first exercise session. MET level is more
of a measurement of workload, not an accurate measurement of an
individual’s response to exercise. Respiratory rate is not used to
prescribe exercise intensity.

C118 C8 II
Explain that this is normal and that the stepping was a newborn reflex
that has gone away.
The mother probably saw the neonatal stepping reflex which is normal
in a newborn but is not exhibited in the older infant probably because
of anthropomorphic factors and neural maturation. The age this reflex
typically disappears is 2 to 3 months of age. In most children pull-to-
stand emerges at 8-9 months while unassisted standing and walking
occurs 10-15 months. Stepping at this age is not the result of a reflex.

C119 C1 I
35-44 mm.
Average AROM is approximately 35 to 50mm. However, only 25-
35mm of opening between the teeth is required for normal everyday
activity. The TMJ's are considered hypomobile if less than 25mm of
opening is achieved. Hypermobility would include values >50 mm.

C120 C1 III
Medial forefoot and rearfoot varus posting
Forefoot varus is a deformity such that the forefoot is in a position of
inversion in relation to the rearfoot when the subtalar joint is in the
neutral position. Forefoot varus results in an abnormal gait pattern
when abnormal compensatory subtalar joint pronation appears,
allowing the medial metatarsal heads to contact the ground. Excessive
calcaneal eversion, talar adduction, and talar plantar flexion occur.
Correction of forefoot varus with excessive subtalar pronation can be
achieved with a customized orthosis. A rearfoot varus post corrects
the excessively everted calcaneus closer to neutral. The medial
forefoot post (medial wedge) allows weight bearing on metatarsal
heads.

C121 C1 II
Pronator teres syndrome
All of the above special tests are used to determine neurological
compromise of the lower trunk and brachial plexus. Special tests to
rule out pronator teres syndrome are: 1) passive supination to
elongate the pronator which is tight; this would compress the nerve at
that level, and 2) active resistance of pronation which would compress
the nerve as it courses through the pronator muscle belly.

478026779.doc
46 Answers

C122 C7 III increased fatigability, and decreased neuromuscular coordination and


Treat the patient, and bill for the 20 minute session given. balance. High variability exists for maximal heart rates in persons over
The physical therapist must deal with the principles of beneficence 65 years of age; the HR reserve method (Karvonen formula) and RPE
(the duty to foster the patient's interests), non-maleficence (the duty is recommended.
not to harm the patient) and justice (the duty to distribute services MET levels vary considerably from individual to individual and cannot
equitably). The therapist is also bound by Principles 4 and 7 of the be relied on; 8 METs represents slow jogging and is not appropriate
APTA's Code of Ethics. Principle 3: Physical therapists accept for this individual.
responsibility for the exercise of sound professional judgment and
Principle 5: Physical therapists seek remuneration for their services C128 C4 II
that is deserved and reasonable. The Guide for Professional Conduct Discontinue UBE exercise; use massage and active assistive ROM.
farther interprets this principle in Principle 7.1 .a Business and This patient presents with the classic signs and symptoms of early
Employment Practices. Physical therapist's professional practices and stage Complex Regional Pain Syndrome, Type I (reflex sympathetic
their adherence to ethical principles shall take precedence over dystrophy). Edema can be successfully managed with a combination
business practices. Provisions of services for personal financial gain of elevation, massage and compression bandaging. Active assistive
rather than for the need of the individual receiving their services are ROM is important to maintain range. Immobilization through splinting
unethical. Fees for physical therapy services should be. reasonable is contraindicated. Ice may be helpful for pain relief; however, it will
for the service performed, considering the setting in which it is not reduce the pitting edema and is not the best course of action.
provided, practice costs in the geographic area, judgment of other
organizations, and other relevant factors.

C123 C5 III
Large electrodes, widely spaced.
Two reasons to place a largo electrode on a largo muscle are to: 1)
stimulate a large number of muscle fibers and 2) reduce the current
density generated under each electrode making the current more
comfortable to the patient. A wide interelectrode spacing will allow the
current to stimulate deep muscle fibers.

C124 C2 III
Supine, trunk in midline with small pillow under the scapula, arm
extended on supporting pillow, and a small towel roll under the knee.
In the upper extremity, spasticity is typically strong in scapular
retractors, shoulder adductors, depressors and internal rotators; elbow
flexors and forearm pronators; wrist and finger flexors. In the lower
extremity, spasticity is typically strong in hip and knee extensors,
pelvic retractors, hip adductors and internal rotators; hip and knee
extensors; plantar flexors. In the trunk, lateral flexors are typically
spastic. Supine with a small pillow under the scapula, arm extended
and small roll under the knee represents the best choice to control for
the expected spasticity.

C125 C2 I
Somatosensory integrity.
The term somatosensation refers to conscious relay pathways for
discriminative touch, conscious proprioception, fast pain and
discriminative temperature. Sensory examination must rule out vision
in order to establish the reliability of sensory testing.
Vestibular/visual/somatosensory integration can only be established
by a series of tests that include both eyes open and eyes closed, flat
and compliant or moving surfaces (see Clinical Test for Sensory
Interaction in Balance, Shumway-Cook and Horak).

C126 C3 II
Congestive heart failure.
An S3 heart sound (ventricular gallop) is indicative of diastolic overload
of the ventricles and decreased ventricular compliance. It is the
hallmark sign of congestive heart failure (left-sided failure) and can be
heard during early diastole after S2 Murmurs (aortic ejection sound,
and pulmonic ejection sounds) are indicative of aortic and pulmonary
valve dysfunction. A pericardial friction rub is heard with pericarditis.

C127 C3 II
HR reserve formula and Ratings of Perceived Exertion.
Expected changes in the elderly include a lower VO2 max which is
evidenced by a lower maximal HR, decreased muscle strength with

478026779.doc
Answers 47

C129 C8 II following positive special tests-apprehension, augmentation, and


Poor light adaptation. relocation.
Poor light adaptation (e.g. going from well lit outside stairs to a dark
inside environment) is a characteristic finding in the elderly. C135 C5 III
Presbyopia (loss of accommodation and near vision) results in reading Readjusting the harness and continuing with the treatment.
difficulties. Safely training should include a suggestion to leave a night Readjusting the cervical harness angle to increase the force at the
light on. back of the skull (occipital protuberance) and lessen the force on the
mandible would decrease the pressure on the TMJ and make the
C130 C6 III treatment more comfortable for the patient.
Sliding board.
This patient does not have the required strength to stand against C136 C6 II
gravity. A sliding board transfer allows the patient to perform the Reposition the height of the shelf and items to below shoulder height.
transfer independently. A dependent 1-man squat transfer (football Workstations should be designed to accommodate the persons who
transfer) is performed with the patient’s arms around the therapist's actually work on the job. Workstations should be easily adjustable and
waist and head down with legs flexed and off the ground. The designed to be comfortable for the worker.
therapist does all the work in lifting and pivoting the patient. This is not
appropriate for this patient. Stand-by assist is equally inappropriate.

C131 C6 III
Lift the front caster and ascend in a wheelie position
Curbs are ascended in the wheelie position (front casters lifted and
moving first). Using momentum from several pushes helps to assist
elevation. Patients can descend backwards but must use a tucked
forward lean position in order to prevent falling backward or can
descend forward using a wheelie position.

C132 C3 II
Decreased bone density.
While many of the choices are sequelae to long term systemic steroid
use, the only one that is a contraindication to percussion would be
decreased bone density, as a rib fracture might be more possible. An
increased blood pressure, probably higher than that reported in the
scenario, might be a contraindication to postural drainage. It is not a
contraindication to percussion. Barrel chest is seen in patients with
emphysema not asthma.

C133C1 II
A Pancoast tumor.
A Pancoast tumor is an apical tumor that is typically found in
conjunction with a smoking history. The clinical signs and symptoms
can be confused with neurovascular compromise at the level of
thoracic outlet. The patient's smoking history, rapid onset of clinical
signs and symptoms, and pleuritic pain is highly suspect of an apical
tumor. The pain reported is not typical of Guillain-Barré syndrome.
Thoracic outlet syndrome does not typically present bilaterally.
Foraminal stenosis in the midcervical spine will not cause weakness in
the hand intrinsics. The smoking history and hoarse voice coupled
with the other symptoms make this a “red flag” situation and
suspicions of a tumor must be reported immediately.

C134
Atraumatic shoulder instability.
The patient's signs and symptoms describe an atraumatic instability of
the shoulder. A patient with a rotator cuff lesion would present with
pain and weakness after eccentric load; protective shoulder hike may
be present; weakness of abduction or rotation or both with AROM;
during PROM, pain would be elicited if impingement occurs; pain and
weakness on abduction and lateral rotation; resisted isometric
movements with a positive drop arm, and empty can test. An
impingement problem would present with positive Neer and Hawkins-
Kennedy impingement tests. The traumatic anteriorly dislocated
shoulder patient would present with apprehension and decreased
ROM in abduction and lateral rotation with AROM; muscle guarding
and decreased ROM in apprehension position with PROM; resisted
isometric tests produce pain in abduction and lateral rotation, with the

478026779.doc
48 Answers

C137 C2 III A contoured foam seat that accommodates to the patient's body
D2 flexion. contours and provides an adequate seat base and lateral support to
The supraspinatus, infraspinatus, teres minor, and subscapularis stabilize the scoliosis provides the best option. A sling seat tends to
comprise the rotator cuff and function to provide dynamic stability to increase asymmetries. Increasing the seat depth encourages a
the shoulder. The supraspinatus assists in abduction while the others posterior pelvic tilt and increases kyphosis. A firm seat with lateral
act during abduction to compress the humeral head into the glenoid knee positioners does not address her trunk problems.
fossa. D2 flexion is the optimal pattern because all the rotator cuff
muscles are activated. In Dl flexion the infraspinatus and teres minor C144 C6 III
are activated but not the supraspinatus. Both Dl and D2 extension Onto her walker and one leg, tuck her pelvis by extending the upper
emphasize internal rotation. trunk, and swing her other leg through.
The correct sequence for using a reciprocating gait orthosis (RGO) is
C138 C6 II to 1) shift weight onto one leg, 2) tuck the pelvis by extending the
Cushion heel with a rearfoot valgus post. upper thorax, 3) press on the crutches or walker; and 4) allow the
This patient is presenting with symptoms of plantar fasciitis and heel other leg to swing through.
pain. A cushion heel with medial forefoot and rear-foot posts that
promote a neutral foot position is the best choice for intervention. A
UCBL insert is prescribed to realign a flexible flat foot. A metatarsal
pad is used to transfer stress from painful metatarsal heads. C145 C7 III
Involve him in goal setting and have him participate in structuring the
C139 C3 II training session.
2+ pulses. An andragogical approach is best. He is an adult learner who should
A bruit is a swishing sound that occurs in the presence of narrowing of be allowed to share in the responsibility for planning his learning
an artery. It is a characteristic finding of PVD. Pulses may be experience. The therapist should help clarify the problem, structure
decreased (l+/diminished or 0/absent); 2+ is normal. the learning environment, and provide necessary resources. Passing
the buck or transferring responsibility to another is not appropriate.
C140 C4 II
Hepatitis B. C146 C7 III
Healthcare workers are most likely to contract hepatitis B (estimated The activities of the non-rehab group were not properly monitored and
incidence 300,000 new acute cases in U.S. each year). Transmission may account for these results.
is through exposure to blood and blood products and infected body To ensure adequate control, the researcher should attempt to remove
fluids. Hepatitis A has a much lower reported incidence (35,000 new the influence of any variable other than the independent variable in
cases each year) and is transmitted primarily through the fecal-oral order to evaluate its effect on the dependent variable. In this study, the
route and contaminated food or water. HIV and tuberculosis also have investigator did not adequately investigate the usual activities of the
lower incidences. HIV has a similar route infection as hepatitis B while control group. The small number of subjects may also have
tuberculosis is an airborne infectious disease. See Table 5.1 Standard contributed to lack of significance.
Precautions.
C147 C6 II
C141 C4 II Lateral lean toward the right.
Rheumatoid arthritis. Lateral trunk bending toward the orthotic stance leg can result from 1)
Rheumatoid arthritis (RA) is characterized by morning stiffness, pain, excessive height of the medial upright, 2) weak gluteus medius
and relatively symmetric joint involvement. Laboratory abnormalities in (Trendelenburg gait), 3) abduction contracture, or 4) a short leg.
RA include: positive serum rheumatoid factor (RF) and elevated Posterior trunk bending is associated with a weak gluteus maximus
erythrocyte sedimentation rate (ESR). Articular and extrarticular while anterior trunk bending is associated with a weak quadriceps.
manifestations include weight loss, malaise, nodulosis and vasculitis.
Synovial fluid analysis reveals elevated white blood cell count and C148 C6 II
protein count. Osteoarthritis (OA) or degenerative joint pain, and A solid ankle AFO
fibromyalgia both produce pain but do not produce the laboratory A solid ankle AFO is indicated to control foot drop in the presence of
findings reported above or nodulosis. Systemic lupus erythematosus spasticity. A grade of 2 (Modified Ashworth Scale) indicates marked
(SLE) is an immunologic disorder characterized by inflammatory increase in muscle tone throughout the range, but the affected part
lesions in multiple organ systems. It is diagnosed by client history can be moved. A dorsiflexion assist and spiral AFO are
(multiple organ involvement, especially of skim, joints, serous contraindicated as they fail to adequately control spasticity and can
membranes), systemic symptoms (fever, malaise, fatigability, etc) and actually increase spasticity. An anterior or dorsiflexion stop limits
appearance of skin rash (erythema). ESR is elevated in patients with dorsiflexion and can be used with weakness of the plantarflexors (not
SLE, however, nodulosis and joint malformations are not expected. a problem in this case).

C142 C6 I C149 C8 II
Prosthetic knee set too far anterior to the TKA line. Delay in achieving developmental milestones.
In order to increase stability of the knee, the prosthetic knee is The 12-15 month-old child should be ambulating. At 24 months lack of
normally aligned 'posterior to a line extending from the trochanter to ambulation is indicative of developmental delay. Other developmental
the ankle (TKA line). A knee set anterior to the TKA line will buckle milestones include: sitting 6 mo. and creeping, pull-to-stand and cruise
easily. The gluteus medius contributes to stability during stance, at 8-9 months.
primarily lateral stability. An extension aid assists knee extension
during the latter part of swing phase. C150 C1 III
Transverse processes of T8.
C143 C6 III The axis of motion for the mid-thoracic vertebrae is above the spinous
Contoured foam seat. processes and below the transverse processes. Therefore, if down-

478026779.doc
Answers 49

gliding/closure of T7-8 vertebral segment is required, the therapist's to increase abnormal tone in those muscles. The problem is a swing
hand placement should be at the transverse process of T8 or the phase deficit; downward compression during stance is inappropriate
spinous process of T7. for this problem.

C151 C1 II C156 C2 II
Thoracic outlet syndrome XII
Thoracic outlet syndrome (TOS) is a neurovascular compression The hypoglossal (C.N. XII) controls the movements of the tongue.
syndrome affecting the upper quadrant. Due to the lack of a good Ipsilateral wasting and the deviation to the ipsilateral side on
definitive objective test to confirm the presence of TOS, the diagnosis protrusion are indicative of damage. Involvement of the
is a clinical one largely reached by exclusion. Ulnar nerve entrapment glossopharyngeal (C.N. IX) results in slight dysphagia, loss of taste in
typically presents with nocturnal numbness, never having sensory loss the posterior third of the tongue and loss of gag reflex. Involvement of
in the proximal or middle portions of the forearm and no atrophy of the accessory (C.N. XI) results in minor problems in deglutition and
intrinsic muscles innervated by the median nerve in the thenar emi- phonation along with weakness in ipsilateral shoulder shrugging.
nence. Carpal tunnel syndrome presents with thenar atrophy and Involvement of the vagus (C.N. X) results in dysphagia, hoarseness
sensory loss in the first two digits differentiated from TOS by and paralysis of the soft palate.
electrophysiologic studies indicating distal and not proximal
compression. Pronator teres syndrome symptoms arise primarily at C157 C3 II
the elbow with radiation into the radial aspect of the hand and Inferior wall.
numbness that extends into the median nerve distribution. With ST elevation and T wave inversion on the ECG are the hallmarks of
passage of time atrophy of the thenar muscles will occur. myocardial infarction. When they occur in leads II, III, and AVF, it is
most likely an inferior infarct. Lateral wall infarcts produce similar
C152 C5 II changes in I, AVL, V5 and V6; anterior wall infarcts produce changes
A vertebral artery test. in V1, V2, V3 and V4; posterior wall infarcts produce changes in V1
The vertebral artery test checks the integrity of the blood flow through and V2 with tall R and T waves and ST depression.
the artery in the cervical region. Since the patient is experiencing
symptoms of circulatory disturbance and a unilateral pull could C158 C4 I
compress the left cervical structures, the vertebral artery test is an All values are abnormal.
appropriate screening test. The test consists of passively placing the The hematocrit value is abnormally low; women average 42% with a
patient's head in extension and side flexion. Then the head and neck normal range from 37% to 47%. The hemoglobin value is also
is slowly rotated to the laterally flexed side and held for 30 seconds. abnormally low; women average 12-to 16-g/100 ml of blood. The low
Some of the positive signs may be syncope, lightheadedness, values are most likely doc to blood loss. The serum WBC (leukocytes)
nystagmus or visual disturbances. Lhermitte’s sign is pain down the is abnormally high; normal values are 5,000 10 10,000/mm³. The
spine and into the upper or lower limbs with passive flexion of the elevated count (greater than 10,000) indicates acute infection.
neck. It is used to identify dysfunction of the spinal cord associated
with upper motor neuron lesions and is typically positive in MS. C159 C4 II
Adson's maneuver is a test for thoracic outlet syndrome. The Dermatitis
Oppenheim test involves running a fingernail along the crest of the This patient is exhibiting symptoms of contact dermatitis. Primary
tibia: a positive lest is the same as a positive Babinski treatment is removal of the offending agent (in this case the total
C153 C2 III contact prosthesis) and treatment of the involved skin with lubricants,
Ambulation using bilateral AFOs and canes topical anesthetics, and/or steroids. He may require a thin sock if the
A spinal cord lesion at the level of L2 is considered a lower motor problem does not resolve. Cellulitis is a suppurative inflammation of
neuron injury (cauda equina injury). Intact movements include hip the dermis and subcutaneous tissues frequently accompanied by
flexion, hip adduction, and knee extension. This patient can be infection. Impetigo is a staphylococcus infection with small macules
expected to be a functional ambulator using bilateral ankle-foot (unraised spots) or vesicles (small blisters). Herpes zoster is a viral
orthoses and crutches or canes. For some community activities, the infection with red papules along the course of a nerve or dermatome.
patient may elect to use wheelchair for convenience and energy
conservation but is not expected to be a full-time wheelchair user. C160 C4 III
Orthotic bracing of the knees is not needed. Surgical repair
A stage IV pressure ulcer is characterized by full thickness skin loss
C154 C2 I with extensive destruction, tissue necrosis, or damage to muscle,
A series of letters traced on the hand. bone or supporting structures. Surgical repair is indicated for patients
Graphesthesia is the ability to recognize numbers, letters, or symbols with extensive and chronic ulcers. The other interventions are not
traced on the skin. Barognosis is the ability to recognize different viable options to resolve this problem.
weights placed in the hand using identically shaped objects.
Pallesthesia is the ability to recognize vibratory stimuli, i.e. a vibrating
tuning fork placed on a bony prominence. Stereognosis is the ability to C161 C6 II
recognize different objects placed in the hand and manipulated. A circumducted gait.
During testing, vision is occluded. Circumduction is a compensation for weak hip flexors or an inability to
shorten the leg (weak knee flexors and ankle dorsiflexors). Hip hiking
C155 C2 III can also compensate for an abnormally long leg (lack of knee flexion
Light resistance to forward pelvic rotation during swing. and dorsiflexion). Excessive hip flexion is a compensation for foot
An elevated and retracted pelvis is a common problem during gait for drop. Forward trunk lean and backward trunk lean are stance phase
many patients recovering from stroke. Providing light resistance to deviations that compensate for quadriceps weakness and glutens
forward pelvic rotation actively engages those muscles and maximus weakness respectively.
reciprocally inhibits the spastic retractors. Providing anterior directed
pressure during swing or resistance to pelvic elevation may only serve C162 C6 III

478026779.doc
50 Answers

Icing and massage to the residual limb Neocerebellum.


This patient is experiencing phantom pain, a common occurrence The neocerebellum (cerebrocerebellum) controls ipsilateral limb
seen in an many as 70% of patients. Treatment interventions can movements, ensuring coordination (adequate force, direction, extent
include icing, pulsed ultrasound, TENS, or massage. Medical of movement, ordering and timing). The spinocerebellum and vermis
interventions include injections and surgical procedures (rhizotomy, control synergistic action of axial and girdle muscles (postural stability)
neurectomy). Prolonged inactivity and bedrest is contraindicated. while the vestibulocerebellum controls equilibrium responses and
Hyaluronidase is indicated for edema reduction. head and eye muscles.
C170 C3 II
C163 C7 III Higher heart rate and arterial blood pressure
The validity of the study was threatened with the introduction of Dynamic exercise facilitates circulation while isometric (static)
sampling bias. exercise hinders blood flow, producing higher heart rates and arterial
The investigator used a sample of convenience and therefore blood pressures. The Valsalva maneuver which accompanies breath
introduced systematic sampling error (a threat to validity). Random holding produces increased intrathoracic pressure, which in turn
selection of subjects would improve the validity of this study. hinders normal venous return to the heart. Breath holding is more
Generalization to a group of patients with chronic ankle sprain cannot likely with isometric exercise but is not always present.
be made. The Hawthorn effect refers to the influence the subject’s
knowledge of participation in the experiment had on the results of the C171 C4 III
study. Ice, elevation and splint for the limb.
Hemarthrosis (bleeding into joint spaces) is associated with swelling,
C164 C3 III joint pain, and decreased ROM and movement. Optimal treatment
Left sidelying with the head of the bed in the flat position. involves pain management along with rest, ice, elevation, and
In order to match perfusion and ventilation, the therapist need to place functional splinting. Weightbearing and exercise during an acute
the unaffected side in a gravity dependent position or that of left bleed are contraindicated.
sidelying. While prone and supine with the head of the bed in the flat
position might helpful, these positions would not be as likely to show C172 C8 II
as much improvement as left sidelying, The use of the Trendelenburg Refer him to his physician as the therapist suspects a small stroke.
position is inappropriate. The presence of focal signs (incoordination, anomia) with cognitive
signs (memory loss) is indicative of impaired brain function and may
C165 C1 I be the result of small strokes. This is the most likely choice given the
A lesion of spinal accessory nerve. spotty symptoms he presents with their sudden onset. Senile
Rotary winging occurs when the inferior angle of one scapula is dementia, Alzheimer’s type, can include some of the same symptoms
rotated farther from the spine that the inferior angle of the other but the onset is gradual and the course is typically slowly progressive.
scapula. Although this type of winging could be found with all of the The reporting of these findings to the primary physician should not be
above answers, the shoulder drooping and inability to shrug the delayed. Further diagnostic workup is indicated. Hospital admission
shoulder is secondary to a lesion of the spinal accessory nerve and anesthesia can cause temporary cognitive difficulties (delirium)
(cranial nerve XI) which innervates the trapezius muscle. but these should not persist with discharge home.

C166 C5 III C173 C6 III


Low detection sensitivity with recording electrodes placed closely Consume a carbohydrate before or during the race to avoid
together. hypoglycemia.
By initially placing the electrodes close together, the therapist During exercise of increasing intensity and duration, plasma
decreases the likelihood of detecting undesired motor units from concentrations of insulin progressively decrease. Exercise-induced
adjacent active muscles (crosstalk). By setting the biofeedback hypoglycemia is the likely result. Hypoglycemia can also occur up to
sensitivity (gain) low the therapist would decrease the amplitude of the 4-6 hours after exercise. To counteract these effects, the individual
signals generated by the hypertonic muscles and keep the EMG may need to reduce his insulin dosage or increase carbohydrate
output from exceeding a visual and/or auditory range (scale). intake before or after running. Consuming a carbohydrate product
before or during the race will have a preventive modulating effect on
C167 C2 II hypoglycemia.
Left posterolateral medulla or pons.
A lesion in the posterolateral medulla or lower pons causes mixed C174 C6 III
sensory loss (described in the question). Pain and temperature are Supracondylar/suprapatellar suspension.
affected while discriminative touch and proprioception are not (the The short amputation limb benefits from supracondylar/suprapatellar
medial lemniscus is not involved). Sensory loss will be completely (SC/SP) suspension. The medial, anterior and lateral walls extend
contralateral only after the discriminative sensory tracts are crossed in above the femoral epicondyles and patella. Patients may experience
the upper midbrain. problems with kneeling with the high anterior wall. The thigh corset is
indicated for individuals with sensitive skin on the amputation limb.
C168 C2 II Syme’s suspension is indicated for the patient with ankle
Drowsiness and muscle weakness. disarticulation.
Baclofen, used in the management of spasticity, can produce CNS
depression (drowsiness, fatigue, weakness, confusion, vertigo, C175 C6 I
dizziness, and insomnia), occurring in less than 10% of patients. Instruct the PTA to have the patient sit down and utilize mental
Additional adverse effects can include hypotension and palpitations, practice of the task.
and urinary frequency. Vomiting, seizures, and coma are signs of Mental rehearsal (mental practice) is the best strategy to have the
overdosage. patient learn the correct sequence. In the nonweightbearing position,
the patient’s anxiety is lessened, leaving her free to concentrate on
C169 C2 II the task at hand. The PT should provide appropriate guidance to the

478026779.doc
Answers 51

PTA but not necessarily take over care. Lack of understanding about
the gait sequence rather than balance difficulties seem to be the major
problem. Distributed practice with long rest times does not address
the main difficulty.

C176 C1 I
Lateral.
The therapist can observe anterior slippage of one vertebral body on
another best using the lateral view. The oblique view is most effective
when evaluating the integrity of the pars interarticularis. The “scotty
dog with a collar” indicates that the pars interarticularis is fractured
and spondylolysis, a precursor of spondylolisthesis, is present.

C177 C5 III
Each side allotting five minutes for each section.
The total treatment area is too large for the 5 cm2 sound head to
produce adequate tissue heating. Moving the transducer too fast to
cover both sides adequately in the allotted time does not allow
sufficient time for the acoustic energy to produce heat. Sonating the
two areas independently will allow more time for the tissue
temperature to rise during the treatment time in each area. Two
minutes is too brief to produce sufficient tissue heating.

478026779.doc
52 Answers

C178 C5 II C185 C6 III


Transcutaneous electrical stimulation Stand in front and slightly to the right side.
All electrical stimulation devices are contraindicated when a patient The correct guarding position is to stand in front and slightly to the
has a demand-type pacemaker. The electrical signals could interfere involved side (the light side in this case). During ascent, the therapist
with the rhythmic signals of the pacemaker. should stand behind and slightly to the involved side.

C179 C2 II C186 C6 III


Vestibular deficiency. A ramp with a slope of 1:12 with a level landing at the top of at least
The clinical test for sensory integration in balance using dynamic 60 by 60 inches.
posturography testing is positive for vestibular deficiency with loss of A ramp should have a slope of 1:12 (for every inch of vertical rise, 12
balance on conditions 5 and 6. Patients who are surface dependent inches of ramp is required). This equals a grade of 8%; a grade of
(somatosensory) have difficulties with conditions 4, 5, and 6. Patients 12% is too steep. A level landing at top of at least 60 by 60 inches is
who are visually dependent have difficulties with conditions 2, 3, and required to allow adequate wheelchair turning radius. Minimum
6. Sensory selection problems are evident with loss of balance on clearance width for doorways is 32 inches; 36 inches is ideal. A
conditions 3-6. minimum width of 40 inches is too wide and would require a custom
built door. The outside door should open in.
C180 C2 II
Loss of the hippocampus and declarative memory function. C187 C2 III
Declarative memory refers to conscious, explicit, or cognitive memory. Increasing the limits of stability and improving center of pressure
It is a function of the cerebral cortex, and the hippocampus. alignment.
Procedural memory (nonconscious memory or implicit memory) refers The patient with Parkinson’s disease exhibits significant balance
to the recall of skills and habits, and emotional responses. It is the impairments including loss of postural reflexes, decreased limits of
result of integrated action of the frontal cortex (neocortex), thalamus, stability, flexed, stooped posture that alters the center of pressure in
and striatum of the basal ganglia. an anterior direction, freezing, and orthostatic hypotension. Platform
balance training should work toward improving the limits of stability
C181 C3 II and center of pressure alignment (the patient should focus on
2 daily sessions of 30 minutes at 40-70% VO2 max reducing anterior displacement).
This individual is obese (body mass index of 30 kg/m2 or more) and
will benefit from exercise to increase energy expenditure and diet to C188 C3 II
reduce caloric intake. The initial exercise prescription should utilize Sinus rhythm with upsloping ST segment depression.
low intensity with longer duration exercise. Splitting the training into 2 The patient is exhibiting sinus rhythm with upsloping ST segment
sessions each day is a good choice. The goal is to work toward depression. An abnormal ECG response is defined as greater than or
bringing the target heart rate into a suitable range. Obese individuals equal to 1.0 mm of horizontal or downsloping depression at 80 msec
are at increased risk of orthopedic injuries and require close beyond the J point (Source: American College of Sports Medicine,
monitoring. ACSM’s Guidelines for Exercise Testing and Prescription, 6th ed.,
2000). Associated clinical signs suggestive of myocardial ischemia
C182 C3 II include dyspnea and angina. Following a review of this patient’s
Compress with one hand to a depth of one third to one half the depth exercise responses coupled with the ECG findings, the therapist
of the chest at a rate of 100/min. correctly determines the exercise session does not have to be
The correct procedure is to use one hand to compress the infant’s terminated. The therapist should continue to closely monitor the
chest to one third or one half its depth. Chest compression should be patient’s responses.
at a rate of 100/min. Previous guidelines called for depression of the
sternum ½ to 1 inch using 2 fingers. (Source: American Heart C189 C8 III
Association, Revised CPR Guidelines, December 2005). Alternate the side of the wheelchair power control.
Alternating the side of the power control will help keep upper extremity
C183 C4 II activity and sitting in the wheelchair more symmetrical. Trunk rotation
Hypokalemia. and extension exercises will not reduce scoliosis. Lateral postural
Hypokalemia, decreased potassium in the blood, is characterized by supports may be indicated. A reclining seat back is not.
the above signs and symptoms. Other possible symptoms include
respiratory distress, irritability, confusion or depression, and GI C190 C1 II
disturbances. Hyperkalemia is excess potassium in the blood. Facet syndrome.
Hyponatremia is decreased sodium in the blood, and hypocalcemia is A facet syndrome presents itself with localized pain. Cervical
decreased calcium in the blood. These conditions cannot produce this radiculopathy presents with arm pain in the dermatomal distribution
battery of symptoms. Refer to Chapter 6 for signs and symptoms of and increased pain by extension and rotation or side flexion. Cervical
these other imbalances. strain presents with pain on activity or when the muscle is on stretch.
Cervical disc herniation has a dermatomal pain distribution with an
C184 C8 II increase of pain on extension, and pain on flexion may either increase
Coronary artery disease. or decrease (most common).
Lack of oxygen to the brain is the-most probable cause of her fainting.
Heart failure with dyspnea and hyperventilation can decrease cerebral C191 C6 II
blood flow by as much as 40%. Postprandial hypotension (a drop in Use job rotation during the workday.
BP after a meal) and hypoglycemia can also cause syncope, but Administrative controls reduce the duration, frequency, and severity of
would not likely cause dyspnea and hyperventilation. Seizures would exposures to ergonomic stressors. Job rotation reduces fatigue and
present with additional clinical features (e.g. an olfactory or visual stress by rotating the worker to jobs that use different muscle-tendon
aura, tongue biting, motor twitching, etc.). groups during the workday. The other choices represent either
engineering or educational interventions.

478026779.doc
Answers 53

C192 C5 IIII
5-second ramp up, 5-second stimulation, 5-second ramp down.
A relatively long ramp up time over a 5-second period is used to
minimize stimulating the muscle too quickly and increasing the
spasticity. The ramp down time has no effect on spasticity.

C193 C2 II
Provide consistent feedback using a blocked practice schedule.
Early learning should focus on consistent feedback given after every
trial to improve initial performance. A blocked practice schedule with
repeated practice of the same skill will also reinforce early learning.
Variable feedback schedules (summed or bandwidth) and variable
practice schedules (serial and random) are indicated for later learning
to improve retention. Using only guided movement is contraindicated
in this case since it minimizes active participation and active learning.

C194 C2 II
Anterior cerebral artery syndrome.
These clinical manifestations are consistent with anterior cerebral
artery syndrome. Patients with middle cerebral artery syndrome
demonstrate the opposite findings, greater involvement of the arm
than leg. Patients with posterior cerebral artery syndrome demonstrate
primary involvement of the visual cortex (contralateral homonymous
hemianopsia) along with dyslexia (difficulty reading), prosopagnosia
(difficulty naming people on sight), and memory defect (temporal lobe
lesion). Patients with basilar artery syndrome demonstrate a
combination of brainstem syndromes along with signs of posterior
cerebral artery syndrome.

478026779.doc
54 Answers

C195 C3 I
Elevation-induced pallor and dependent redness with the extremity in
the gravity dependent position.
Rubor of dependency test is used to assess the adequacy of arterial
circulation by evaluating the skin color changes that occur with first,
extremity elevation (pallor) and then lowering of the extremities
(delayed color changes, redness).

C196 C4 III
Pelvic floor and gentle abdominal exercises for the first 4-6 weeks.
Post-cesarean physical therapy can include postoperative TENS,
assisted breathing and coughing techniques, and gentle abdominal
exercises with incisional support provided by a pillow. Pelvic floor
exercises are also important since hours of labor and pushing are
typically present before surgery. Vigorous exercise is contraindicated
for at least 6 weeks.

C197 C8 II
Depression.
Many elderly experience social isolation and major depression. All of
the above clinical manifestations can be indicative of depression.
Additional signs and symptoms include: anxiety, crying spells, Jack of
self-confidence, low self-esteem and self-reproach, negative
expectations (hopelessness, helplessness, increased dependency);
suicidal thoughts and delusions. While depression can be associated
with the other choices, there is no evidence in this case that they are
contributing factors.

C198 C6 III
Adding horizontal grab bars positioned at 45 inches.
Horizontal grab bars should be positioned at an optimal height of 33-
36 inches. The toilet seat should be raised to a height of 17-19 inches.
Minimum clearance width for doorways is 32 inches; 36 inches is
ideal.

C199 C6 II
Reduce peak rise of the pelvis.
Lateral pelvic tilt in the frontal plane keeps she peak of the sinusoidal
curve lower than it would have been if the pelvis did not drop. Lateral
pelvic tilt to the right is controlled by the left hip abductors. Forward
and backward rotations of the pelvis assist the swing leg. The normal
physiologic valgum at the knee reduces the width of the base of
support. Knee flexion at midstance is another adjustment in keeping
the center of gravity from rising too much. All are termed determinants
of gait.

C200 C1 II
Thoracic posture syndrome.
Thoracic posture syndrome presents in people who perform work that
involves a lot of sitting in a kyphosed posture for prolonged periods. A
dorsal shift of disc increases the pressure on the posterior longitudinal
ligament and the dura mater, leading to overload of other dorsal
ligaments and paravertebral muscles. HNP in the thoracic region is
almost always a result of trauma, and usually involves axial
compression (fall on the buttocks) or when carrying a heavy object
with the thoracic spine in flexion. Spondylodiscitis is inflammation of
the intervertebral disc that has a hematogenic cause, which presents
with acute or gradual onset of pain in the back with radiation around
the side and sometimes even into the legs. Traumatic compression
syndrome is due to an axial force acting on a flexed thoracic spine,
most frequently affecting T12, LI and L2. The patient typically presents
with acute immediate pain, with every movement extremely difficult
due to pain.

478026779.doc

Вам также может понравиться